标签: STAT3921

数学代写|随机过程统计代写Stochastic process statistics代考|STAT7004

如果你也在 怎样代写随机过程统计Stochastic process statistics这个学科遇到相关的难题,请随时右上角联系我们的24/7代写客服。

随机过程 用于表示在时间上发展的统计现象以及在处理这些现象时出现的理论模型,由于这些现象在许多领域都会遇到,因此这篇文章具有广泛的实际意义。

statistics-lab™ 为您的留学生涯保驾护航 在代写随机过程统计Stochastic process statistics方面已经树立了自己的口碑, 保证靠谱, 高质且原创的统计Statistics代写服务。我们的专家在代写随机过程统计Stochastic process statistics代写方面经验极为丰富,各种代写随机过程统计Stochastic process statistics相关的作业也就用不着说。

我们提供的随机过程统计Stochastic process statistics及其相关学科的代写,服务范围广, 其中包括但不限于:

  • Statistical Inference 统计推断
  • Statistical Computing 统计计算
  • Advanced Probability Theory 高等概率论
  • Advanced Mathematical Statistics 高等数理统计学
  • (Generalized) Linear Models 广义线性模型
  • Statistical Machine Learning 统计机器学习
  • Longitudinal Data Analysis 纵向数据分析
  • Foundations of Data Science 数据科学基础
数学代写|随机过程统计代写Stochastic process statistics代考|STAT7004

数学代写|随机过程统计代写Stochastic process statistics代考|Waiting time for a gain

Let $\left{X_{i}\right}$ be sequence of i.i.d r.v.s with common distribution $P\left(X_{k}=1\right)=p, p\left(X_{k}=-1\right)=q, p+q=1$ and $S_{n}=X_{1}+\ldots+X_{n}, S_{0}=0$
In gambling terminology $S_{n}, n \geq 1$ is the Peter’s accumulated gain at the end of the $n^{\text {th }}$ trial if Peter and Paul play for unit stakes. Now consider the event $A_{n}=\left{S_{1} \leq 0, S_{2} \leq 0, \ldots, S_{n-1} \leq 0, S_{n}=1\right}$.

Thus, the $n^{\text {th }}$ trial is the first to render Peter’s accumulated gain positive. The event $A_{n}$ is called first visit to $+1$ or the index $n$ is the passage time through $l$ in random walk terminology.
Let $\phi_{n}=P\left(A_{n}\right)$. Define $\phi_{0}=0, \phi_{1}=p$.
If the event holds for $n>1$, then $S_{1}=-1$ and there exists a smallest integer $v$ $<n$ such that $S_{v}=0$. The outcome of the first $n$ trials may be described as follows: (1) At the first trial Peter looses an unit amount. (2) It takes $v-1$ further trials for Peter to reestablish the initial situation. (3) It takes exactly $n-v$ further trials for Peter to attain a positive net gain. These events depend on non-overlapping blocks of trials and are therefore mutually independent, $(2)$ and (3) have probabilities $\phi_{t-1}$ and $\phi_{n-v}$.

Now the event $A_{n}$ occurs iff the events (1) to (3) occcur for some $v<n$. Summing over all possible $1 \leq v \leq n-1$, we get
$$
\phi_{n}=q\left(\phi_{1} \phi_{n-2}+\ldots+\phi_{n-2} \phi_{1}\right)=q \sum_{v=2}^{n-1} \phi_{v-1} \phi_{n-v}
$$
Multiplying both sides by $s^{n}$ and summing $n=2,3, \ldots$, we get
$$
\begin{aligned}
\sum_{n=2}^{\infty} \phi_{n} s^{n} &=q s\left{\sum_{n=2}^{\infty} \phi_{n-1} s^{n-1}\right}^{2} \
&=q s\left{\sum_{n=0}^{\infty} \phi_{n} s^{n}\right}^{2}=q s \Phi^{2}(s)
\end{aligned}
$$
$\left(\right.$ since $\left.\phi_{0}=0\right)$
Also
$$
\sum_{n=2}^{\infty} \phi_{n} s^{n}=\Phi(s)-\phi_{0}-\phi_{1} s=\Phi(s)-p s
$$
where $\Phi(s)$ is the G.F. of $\left{\phi_{n}\right}$.
Equation (3.3) follows from the fact that
$$
\phi_{1} \phi_{n-2}+\ldots+\phi_{n-2} \phi_{1}=(n-1) \text { th term of }\left{\phi_{n}\right}^{*}\left{\phi_{n}\right} .
$$
and
$$
\begin{aligned}
& \sum_{n=2}^{\infty} q\left{\phi_{1} \phi_{n-2}+\ldots+\phi_{n-2} \phi_{1}\right} s^{n} \
=& q s \sum_{n=2}^{\infty}\left{\phi_{1} \phi_{n-2}+\ldots+\phi_{n-2} \phi_{1}\right} s^{n-1}
\end{aligned}
$$
Hence from (3.3) and (3.4), $\Phi(s)-p s=q s \Phi^{2}(s)$.

数学代写|随机过程统计代写Stochastic process statistics代考|Returns to equilibrium

Let $A_{k}$ be the event of equalization of the accumulated number of successes and failures occurs at the $k$ th trial if $S_{k}=0$. Let $u_{k}=P\left(S_{k}=0\right)$. The number of trials is necessarily even and the probability of a return to the origin at the $2 n$th trial is given by
$$
U_{2 n}=\left(\begin{array}{c}
2 n \
n
\end{array}\right) p^{n} q^{n}=(-1)^{n}\left(\begin{array}{c}
-\frac{1}{2} \
n
\end{array}\right)(4 p q)^{n}
$$
The G.F. of $\left{U_{2 n}\right}$ is $U(s)=\sum_{n=0}^{\infty} U_{2 n} s^{2 n}$
$$
=\sum_{n=0}^{\infty}(-1)^{n}\left(\begin{array}{c}
-\frac{1}{2} \
n
\end{array}\right)\left(4 p q s^{2}\right)^{n}=\left(1-4 p q s^{2}\right)^{-\frac{1}{2}}
$$
The first return to (origin) equilibrium,
$$
B_{2 n}=\left[S_{k} \neq 0, \text { for } k=1, \ldots, 2 n-1, S_{2 n}=0\right] .
$$
Let $P\left(B_{2 n}\right)=f_{2 n}$.
Consider two sub-events with $X_{1}=1, X_{1}=-1$ and denote their probabilities by $f_{2 n}^{+}$and $f_{2 n}^{-}$, i.e.
$$
f_{2 n}^{+}=P\left(B_{n} \cap\left(X_{1}=1\right)\right) \text { and } f_{2 n}^{-}=P\left(B_{n} \cap\left(X_{1}=-1\right)\right) \text {. }
$$
Now $f_{2 n}^{-}=q \phi_{2 n-1}$ (because first $2 n-2$ partial sums $X_{2}+X_{3}+\ldots+X_{n} \leq 0$, but the next one is positive)
As before let $\phi_{n}=P\left[S_{1} \leq 0, S_{2} \leq 0, \ldots, S_{n}=1\right]$
Then the G.F. of $\left{f_{2 n}^{-}\right}$is
$$
\begin{aligned}
F^{-}(s) &=\sum_{n=1}^{\infty} f_{2 n}^{-} s^{2 n}=s q \sum_{n=1}^{\infty} \phi_{2 n-1} s^{2 n-1} \
&=q s \Phi(s)=q s \frac{1-\left(1-4 p q s^{2}\right)^{\frac{1}{2}}}{2 q s}
\end{aligned}
$$
By symmetry, $F^{+}(s)=F^{-}(s)$ and hence
$$
\begin{aligned}
\sum_{n=1}^{\infty} f_{2 n} s^{2 n} &=F(s)=F^{+}(s)=F^{-}(s)=1-\left(1-4 p q s^{2}\right)^{\frac{1}{2}} \
(&\left.=1-\frac{1}{U(S)} \text { in general }\right)
\end{aligned}
$$
Hence probability that a return to equilibrium occurs sooner or later will be $$
\begin{aligned}
F(1) &=\sum_{n=1}^{\infty} f_{2 n} \
&=1-(1-4 p q)^{\frac{1}{2}}=1-|p-q| \
&= \begin{cases}2 q & \text { if } p>q \
2 p & \text { if } p<q \
1 & \text { if } p=q=\frac{1}{2}\end{cases}
\end{aligned}
$$
Hence, if $p=q=\frac{1}{2}$ a return to equilibrium is certain.

数学代写|随机过程统计代写Stochastic process statistics代考|Sequential Analysis

An important problem arising in Wald’s sequential analysis is concerned with the random variable $N=N(a, b)$, where $N=\min \left{n \mid S_{n} \leq-b\right.$ or $\left.S_{n} \geq a\right}$ is the first exist time from the interval $(-b, a)$.
We ignore the trivial case $P\left(X_{i}=0\right)=1$.
Let $X_{i}$ are i.i.d. r.v.s and $S_{n}=X_{1}+\ldots+X_{n}$.
Theorem $3.1$ (C. Stein 1947)
$N$ is a proper random variable with finite moments of all order, i.e.
(i) $P(N<\infty)=1$ and (ii) $E(N)^{k}<\infty$ for all $k=1,2, \ldots$ Proof (i) We shall show, more specifically that there exists $A>0$ and
$$
0<\delta<1 \text { independent of } n \text { and } P[N \geq n] \leq A \delta^{n}
$$
Let $C=a+b$ and $r$ be a positive integer.
$$
\text { Let } \begin{aligned}
S_{1}^{}=X_{1}+\ldots+X_{r}, S_{2}^{} &=X_{r+1}+X_{r+2}+\ldots+X_{2 r}, \ldots, \
S_{k}^{*} &=X_{(k-1) r+1}+\ldots+X_{k r}
\end{aligned}
$$

We have, $P[N \geq k r] \leq P\left[\left|S_{1}^{}\right|}\right|}\right|}\right|0$.
If $p=1$, then $E\left(S_{k}^{}\right)^{2}=r E X_{i}^{2}+r(r-1)\left(E X_{i}\right)^{2}$ (since $X_{i}$ ‘s are i.i.d.) Since $E\left(X_{i}^{2}\right)>0, E\left(S_{k}^{}\right)^{2}>C^{2}$ by choosing $r$ large enough. But $p=1 \Rightarrow$ $E\left(S_{k}^{*}\right)^{2} \leq C^{2}$, which is a contradiction. Therefore $p \neq 1$ and $P(N<\infty)=1$. (ii) For $t>0$ and positive integer $k, n^{k}<e^{t n}$ for large $n$,
$$
\sum_{n=m}^{\infty} n^{k} P[N=n] \leq \sum_{n=m}^{\infty} e^{i n} P[N \geq n] \leq A \sum_{n=m}^{\infty}\left(\delta e^{t}\right)^{n}<\infty \text { if } \delta e^{t}<1
$$
Hence
$$
\begin{aligned}
E\left(N^{k}\right) &=\sum_{n=1}^{\infty} n^{k} P[N=n] \
&=\sum_{n=1}^{m-1} n^{k} P[N=n]+\sum_{n=m}^{\infty} n^{k} P[N=n]
\end{aligned}
$$
Definition $3.1 \quad N$ is called a stopping rule if $N$ is a non-negative integer-valued random variable and the event $[N \geq n]$ depends on $X_{1}, X_{2}, \ldots X_{n-1}$ only, i.e. $\lfloor N=n]$ is measurable with respect to $S D\left(X_{1}, \ldots, X_{n-1}\right)\left(X_{1}, \ldots, X_{n-1}\right.$, need not be i.i.d. r.v.s).

数学代写|随机过程统计代写Stochastic process statistics代考|STAT7004

随机过程统计代考

数学代写|随机过程统计代写Stochastic process statistics代考|Waiting time for a gain

让\left{X_{i}\right}\left{X_{i}\right}是具有共同分布的独立同分布 rvs 序列磷(Xķ=1)=p,p(Xķ=−1)=q,p+q=1和小号n=X1+…+Xn,小号0=0
在赌博术语中小号n,n≥1是彼得在结束时的累积收益nth 如果彼得和保罗参加单位赌注,则进行审判。现在考虑事件A_{n}=\left{S_{1} \leq 0, S_{2} \leq 0, \ldots, S_{n-1} \leq 0, S_{n}=1\right}A_{n}=\left{S_{1} \leq 0, S_{2} \leq 0, \ldots, S_{n-1} \leq 0, S_{n}=1\right}.

就这样nth 试验是第一个使彼得的累积收益为正的。事件一个n被称为首次访问+1或索引n是通过时间l在随机游走术语中。
让φn=磷(一个n). 定义φ0=0,φ1=p.
如果活动持续n>1, 然后小号1=−1并且存在一个最小整数在 <n这样小号在=0. 第一场的结果n试验可以描述如下: (1) 在第一次试验中,彼得损失了一个单位金额。(2) 需要在−1进一步的试验让彼得重新建立最初的情况。(3) 精确到n−在进一步的试验让彼得获得正的净收益。这些事件依赖于不重叠的试验块,因此是相互独立的,(2)(3) 有概率φ吨−1和φn−在.

现在事件一个n如果事件 (1) 到 (3) 发生了一些在<n. 总结所有可能的1≤在≤n−1,我们得到

φn=q(φ1φn−2+…+φn−2φ1)=q∑在=2n−1φ在−1φn−在
两边乘以sn和求和n=2,3,…,我们得到

\begin{对齐} \sum_{n=2}^{\infty} \phi_{n} s^{n} &=q s\left{\sum_{n=2}^{\infty} \phi_{n- 1} s^{n-1}\right}^{2} \ &=q s\left{\sum_{n=0}^{\infty} \phi_{n} s^{n}\right}^{ 2}=q s \Phi^{2}(s) \end{对齐}\begin{对齐} \sum_{n=2}^{\infty} \phi_{n} s^{n} &=q s\left{\sum_{n=2}^{\infty} \phi_{n- 1} s^{n-1}\right}^{2} \ &=q s\left{\sum_{n=0}^{\infty} \phi_{n} s^{n}\right}^{ 2}=q s \Phi^{2}(s) \end{对齐}
(自从φ0=0)

∑n=2∞φnsn=披(s)−φ0−φ1s=披(s)−ps
在哪里披(s)是 GF\左{\phi_{n}\右}\左{\phi_{n}\右}.
方程 (3.3) 由以下事实得出:

\phi_{1} \phi_{n-2}+\ldots+\phi_{n-2} \phi_{1}=(n-1) \text { }\left{\phi_{n}\right }^{*}\left{\phi_{n}\right} 。\phi_{1} \phi_{n-2}+\ldots+\phi_{n-2} \phi_{1}=(n-1) \text { }\left{\phi_{n}\right }^{*}\left{\phi_{n}\right} 。

\begin{aligned} & \sum_{n=2}^{\infty} q\left{\phi_{1} \phi_{n-2}+\ldots+\phi_{n-2} \phi_{1}\右} s^{n} \ =& q s \sum_{n=2}^{\infty}\left{\phi_{1} \phi_{n-2}+\ldots+\phi_{n-2} \phi_ {1}\right} s^{n-1} \end{对齐}\begin{aligned} & \sum_{n=2}^{\infty} q\left{\phi_{1} \phi_{n-2}+\ldots+\phi_{n-2} \phi_{1}\右} s^{n} \ =& q s \sum_{n=2}^{\infty}\left{\phi_{1} \phi_{n-2}+\ldots+\phi_{n-2} \phi_ {1}\right} s^{n-1} \end{对齐}
因此从(3.3)和(3.4),披(s)−ps=qs披2(s).

数学代写|随机过程统计代写Stochastic process statistics代考|Returns to equilibrium

让一个ķ是累积成功和失败次数均等的事件发生在ķ审判如果小号ķ=0. 让在ķ=磷(小号ķ=0). 试验次数必然是偶数,返回原点的概率2n审判由

在2n=(2n n)pnqn=(−1)n(−12 n)(4pq)n
女朋友\left{U_{2 n}\right}\left{U_{2 n}\right}是在(s)=∑n=0∞在2ns2n

=∑n=0∞(−1)n(−12 n)(4pqs2)n=(1−4pqs2)−12
第一次回到(原点)均衡,

乙2n=[小号ķ≠0, 为了 ķ=1,…,2n−1,小号2n=0].
让磷(乙2n)=F2n.
考虑两个子事件X1=1,X1=−1并将它们的概率表示为F2n+和F2n−, IE

F2n+=磷(乙n∩(X1=1)) 和 F2n−=磷(乙n∩(X1=−1)). 
现在F2n−=qφ2n−1(因为首先2n−2部分金额X2+X3+…+Xn≤0,但下一个是正数)
和以前一样让φn=磷[小号1≤0,小号2≤0,…,小号n=1]
然后的GF\left{f_{2 n}^{-}\right}\left{f_{2 n}^{-}\right}是

F−(s)=∑n=1∞F2n−s2n=sq∑n=1∞φ2n−1s2n−1 =qs披(s)=qs1−(1−4pqs2)122qs
通过对称,F+(s)=F−(s)因此

∑n=1∞F2ns2n=F(s)=F+(s)=F−(s)=1−(1−4pqs2)12 (=1−1在(小号) 一般来说 )
因此,迟早会恢复平衡的概率为

F(1)=∑n=1∞F2n =1−(1−4pq)12=1−|p−q| ={2q 如果 p>q 2p 如果 p<q 1 如果 p=q=12
因此,如果p=q=12回归平衡是肯定的。

数学代写|随机过程统计代写Stochastic process statistics代考|Sequential Analysis

Wald 序列分析中出现的一个重要问题与随机变量有关ñ=ñ(一个,b), 在哪里N=\min \left{n \mid S_{n} \leq-b\right.$ 或 $\left.S_{n} \geq a\right}N=\min \left{n \mid S_{n} \leq-b\right.$ 或 $\left.S_{n} \geq a\right}是间隔中的第一个存在时间(−b,一个).
我们忽略微不足道的情况磷(X一世=0)=1.
让X一世是 iidrvs 和小号n=X1+…+Xn.
定理3.1(C.斯坦 1947)
ñ是具有所有阶的有限矩的适当随机变量,即
(i)磷(ñ<∞)=1(ii)和(ñ)ķ<∞对所有人ķ=1,2,…证明 (i) 我们将证明,更具体地说,存在一个>0和

0<d<1 独立于 n 和 磷[ñ≥n]≤一个dn
让C=一个+b和r为正整数。

 让 小号1=X1+…+Xr,小号2=Xr+1+Xr+2+…+X2r,…, 小号ķ∗=X(ķ−1)r+1+…+Xķr

我们有,P[N \geq k r] \leq P\left[\left|S_{1}^{}\right|}\right|}\right|}\right|0P[N \geq k r] \leq P\left[\left|S_{1}^{}\right|}\right|}\right|}\right|0.
如果p=1, 然后和(小号ķ)2=r和X一世2+r(r−1)(和X一世)2(自从X一世是 iid) 因为和(X一世2)>0,和(小号ķ)2>C2通过选择r足够大。但p=1⇒ 和(小号ķ∗)2≤C2,这是一个矛盾。所以p≠1和磷(ñ<∞)=1. (ii) 为吨>0和正整数ķ,nķ<和吨n对于大n,

∑n=米∞nķ磷[ñ=n]≤∑n=米∞和一世n磷[ñ≥n]≤一个∑n=米∞(d和吨)n<∞ 如果 d和吨<1
因此

和(ñķ)=∑n=1∞nķ磷[ñ=n] =∑n=1米−1nķ磷[ñ=n]+∑n=米∞nķ磷[ñ=n]
定义3.1ñ称为停止规则,如果ñ是一个非负整数值随机变量,事件[ñ≥n]取决于X1,X2,…Xn−1只有,即⌊ñ=n]是可测量的小号D(X1,…,Xn−1)(X1,…,Xn−1, 不必是 iidrvs)。

数学代写|随机过程统计代写Stochastic process statistics代考 请认准statistics-lab™

统计代写请认准statistics-lab™. statistics-lab™为您的留学生涯保驾护航。

金融工程代写

金融工程是使用数学技术来解决金融问题。金融工程使用计算机科学、统计学、经济学和应用数学领域的工具和知识来解决当前的金融问题,以及设计新的和创新的金融产品。

非参数统计代写

非参数统计指的是一种统计方法,其中不假设数据来自于由少数参数决定的规定模型;这种模型的例子包括正态分布模型和线性回归模型。

广义线性模型代考

广义线性模型(GLM)归属统计学领域,是一种应用灵活的线性回归模型。该模型允许因变量的偏差分布有除了正态分布之外的其它分布。

术语 广义线性模型(GLM)通常是指给定连续和/或分类预测因素的连续响应变量的常规线性回归模型。它包括多元线性回归,以及方差分析和方差分析(仅含固定效应)。

有限元方法代写

有限元方法(FEM)是一种流行的方法,用于数值解决工程和数学建模中出现的微分方程。典型的问题领域包括结构分析、传热、流体流动、质量运输和电磁势等传统领域。

有限元是一种通用的数值方法,用于解决两个或三个空间变量的偏微分方程(即一些边界值问题)。为了解决一个问题,有限元将一个大系统细分为更小、更简单的部分,称为有限元。这是通过在空间维度上的特定空间离散化来实现的,它是通过构建对象的网格来实现的:用于求解的数值域,它有有限数量的点。边界值问题的有限元方法表述最终导致一个代数方程组。该方法在域上对未知函数进行逼近。[1] 然后将模拟这些有限元的简单方程组合成一个更大的方程系统,以模拟整个问题。然后,有限元通过变化微积分使相关的误差函数最小化来逼近一个解决方案。

tatistics-lab作为专业的留学生服务机构,多年来已为美国、英国、加拿大、澳洲等留学热门地的学生提供专业的学术服务,包括但不限于Essay代写,Assignment代写,Dissertation代写,Report代写,小组作业代写,Proposal代写,Paper代写,Presentation代写,计算机作业代写,论文修改和润色,网课代做,exam代考等等。写作范围涵盖高中,本科,研究生等海外留学全阶段,辐射金融,经济学,会计学,审计学,管理学等全球99%专业科目。写作团队既有专业英语母语作者,也有海外名校硕博留学生,每位写作老师都拥有过硬的语言能力,专业的学科背景和学术写作经验。我们承诺100%原创,100%专业,100%准时,100%满意。

随机分析代写


随机微积分是数学的一个分支,对随机过程进行操作。它允许为随机过程的积分定义一个关于随机过程的一致的积分理论。这个领域是由日本数学家伊藤清在第二次世界大战期间创建并开始的。

时间序列分析代写

随机过程,是依赖于参数的一组随机变量的全体,参数通常是时间。 随机变量是随机现象的数量表现,其时间序列是一组按照时间发生先后顺序进行排列的数据点序列。通常一组时间序列的时间间隔为一恒定值(如1秒,5分钟,12小时,7天,1年),因此时间序列可以作为离散时间数据进行分析处理。研究时间序列数据的意义在于现实中,往往需要研究某个事物其随时间发展变化的规律。这就需要通过研究该事物过去发展的历史记录,以得到其自身发展的规律。

回归分析代写

多元回归分析渐进(Multiple Regression Analysis Asymptotics)属于计量经济学领域,主要是一种数学上的统计分析方法,可以分析复杂情况下各影响因素的数学关系,在自然科学、社会和经济学等多个领域内应用广泛。

MATLAB代写

MATLAB 是一种用于技术计算的高性能语言。它将计算、可视化和编程集成在一个易于使用的环境中,其中问题和解决方案以熟悉的数学符号表示。典型用途包括:数学和计算算法开发建模、仿真和原型制作数据分析、探索和可视化科学和工程图形应用程序开发,包括图形用户界面构建MATLAB 是一个交互式系统,其基本数据元素是一个不需要维度的数组。这使您可以解决许多技术计算问题,尤其是那些具有矩阵和向量公式的问题,而只需用 C 或 Fortran 等标量非交互式语言编写程序所需的时间的一小部分。MATLAB 名称代表矩阵实验室。MATLAB 最初的编写目的是提供对由 LINPACK 和 EISPACK 项目开发的矩阵软件的轻松访问,这两个项目共同代表了矩阵计算软件的最新技术。MATLAB 经过多年的发展,得到了许多用户的投入。在大学环境中,它是数学、工程和科学入门和高级课程的标准教学工具。在工业领域,MATLAB 是高效研究、开发和分析的首选工具。MATLAB 具有一系列称为工具箱的特定于应用程序的解决方案。对于大多数 MATLAB 用户来说非常重要,工具箱允许您学习应用专业技术。工具箱是 MATLAB 函数(M 文件)的综合集合,可扩展 MATLAB 环境以解决特定类别的问题。可用工具箱的领域包括信号处理、控制系统、神经网络、模糊逻辑、小波、仿真等。

R语言代写问卷设计与分析代写
PYTHON代写回归分析与线性模型代写
MATLAB代写方差分析与试验设计代写
STATA代写机器学习/统计学习代写
SPSS代写计量经济学代写
EVIEWS代写时间序列分析代写
EXCEL代写深度学习代写
SQL代写各种数据建模与可视化代写

数学代写|随机过程统计代写Stochastic process statistics代考|MTH 3016

如果你也在 怎样代写随机过程统计Stochastic process statistics这个学科遇到相关的难题,请随时右上角联系我们的24/7代写客服。

随机过程 用于表示在时间上发展的统计现象以及在处理这些现象时出现的理论模型,由于这些现象在许多领域都会遇到,因此这篇文章具有广泛的实际意义。

statistics-lab™ 为您的留学生涯保驾护航 在代写随机过程统计Stochastic process statistics方面已经树立了自己的口碑, 保证靠谱, 高质且原创的统计Statistics代写服务。我们的专家在代写随机过程统计Stochastic process statistics代写方面经验极为丰富,各种代写随机过程统计Stochastic process statistics相关的作业也就用不着说。

我们提供的随机过程统计Stochastic process statistics及其相关学科的代写,服务范围广, 其中包括但不限于:

  • Statistical Inference 统计推断
  • Statistical Computing 统计计算
  • Advanced Probability Theory 高等概率论
  • Advanced Mathematical Statistics 高等数理统计学
  • (Generalized) Linear Models 广义线性模型
  • Statistical Machine Learning 统计机器学习
  • Longitudinal Data Analysis 纵向数据分析
  • Foundations of Data Science 数据科学基础
数学代写|随机过程统计代写Stochastic process statistics代考|MTH 3016

数学代写|随机过程统计代写Stochastic process statistics代考|Different Types of Random Walks

(a) Unrestricted Random Walk
In this the elements of transition matrix is given by $p_{i, i+1}=p, p_{i, i-1}=q$, for all integer $i(\ldots,-1,0,1,2, \ldots)$.
If $0<p<1$, the chain is irreducible. Then we have
$$
p_{i j}^{(n)}=P\left(S_{n}=j-i\right)=\left(\begin{array}{c}
n \
(n-j+i) i 2
\end{array}\right) p^{\frac{n+j-i}{2}} q^{\frac{n-j+i}{2}} \text { if } n \text { is even }
$$
$=0$ if $n$ is odd.
and
$$
p_{00}^{(n)}=\left(\begin{array}{c}
n \
\frac{n}{2}
\end{array}\right)(p q)^{n / 2}
$$
The period of the chain is 2 .
It is transient if $p \neq \frac{1}{2}$ and null recurrent if $p=\frac{1}{2}$.

(b) Random Walk with an Absorbing Barrier
In this walk the elements of transition matrix are given by $p_{i, i+1}=p, p_{i, i-1}=q$, $(p+q=1), p_{00}=1$ for all $i \geq 1$.
‘ 0 ‘ is an absorbing state and the remaining states are all transient. $0,-1,-2$, $-3, \ldots$ are condensed into a single absorbing state ‘ 0 ‘.
Let $f_{i 0}^{(n)}=$ Probability of visiting ‘ 0 ‘ from $i$, first time in $n$ steps
$$
=\left(\begin{array}{l}
i \
n
\end{array}\right)\left(\begin{array}{c}
n \
(n-1) / 2
\end{array}\right) p^{(n-i) / 2} q^{(n+i) / 2}
$$
Probability of visiting ‘ 0 ‘ from $i$ ever,
$$
\begin{gathered}
f_{i 0}=\sum_{n} f_{i 0}^{(n)} \text { satisfies difference equations } \
f_{i 0}=p f_{i+1,0}+q f_{i-1,0} \text { for } i>1, f_{10}=p f_{20}+q .
\end{gathered}
$$
Hence solving we get
$$
f_{i 0}=\left{\begin{array}{l}
1 \text { if } p \leq q \
(q / p)^{i} \text { if } p \geq q
\end{array}\right.
$$
(c) Random Walk with Two Absorbing Barries
Here the elements of transition matrix is given by
$$
p_{i, i+1}=p, p_{i, i-1}=q \text { for } 1 \leq i \leq a-1, p_{00}=1, p_{a a}=1 .
$$
$’ 0$ ‘ and ‘ $a$ ‘ are absorbing and remaining states are transient.

数学代写|随机过程统计代写Stochastic process statistics代考|Examples of Random Walks with Absorbing Barriers

Gambler’s Ultimate Ruin Problem
The fortune of a gambler forms a M.C. with transition matrix
$p_{i j}=\left{\begin{array}{l}p \text { if } j=i+1 \ q \text { if } j=i-1 \ 0, \text { otherwise }\end{array}\right.$ and $i=2,3, \ldots, s$
$p_{i j}=\left{\begin{array}{l}1 \text { if } j=1 \ 0 \text { if } j \neq 1\end{array} \quad\right.$ and $i=1$ and $s .$
More explicitly the transition matrix is given by
$$
P=\left[\begin{array}{ccccc}
1 & 0 & 0 & & 0 \
q & 0 & p & 0 & 0 \
0 & q & 0 & p & 0 \
\cdots & \cdots & \cdots & \cdots & \cdots \
0 & 0 & 0 & 0 & 1
\end{array}\right]
$$

数学代写|随机过程统计代写Stochastic process statistics代考|Random Walks

Let $\left{X_{n}, n=0,1,2,3, \ldots\right}$ be a sequence of independent discrete random variables taking integral values only and $S_{n}=X_{1}+X_{2} \ldots+X_{n}(n=0,1,2, \ldots)$. Then the sequence $\left{S_{n}\right}$ is a M.C. whose transition probabilities are given by,
$$
{ }^{(m)} p_{i j}=P\left(S_{m+1}=j \mid S_{m}=i\right)=P\left(X_{m+1}=j-i\right), i, j=\ldots,-2,-1,0,1,2, \ldots
$$
(non-homogeneous random walk).
The chain represents a Random walk of a particle along a straight line, the magnitude of ‘jump’ at time $n$ being given by the random variable $X_{n}$. If $X_{0}$ is denotes the initial position of a particle then its position after $n$ jumps (at time $n$ ) is given by $S_{n}$. When $X_{n}$ ‘s are also indentically distributed, ${ }^{(n)} p_{i j}=p_{j-i}$ where $p_{j}$ $=P\left(X_{n}=j\right.$ ). We have then a homogeneous Random walk (RW). Such Random walks occur in fluctuation theory (sums of discrete or continuous random variables). In classical RW, $P\left(X_{n}=+1\right)=p, p\left(X_{n}=-1\right)=q=1-p$.

In terms of gambling this can be described as follows:
If two gamblers play a series of games in which the probability of a particular player winning is $p$ for each game ( $q=1-p$ is the probability of losing a game). If the player loses he gives one unit of money to his opponent and if he wins he receives one unit from his opponent. If this particular player starts with $x$ units of money and his opponent with $s-x$ units, what is the probability of the player losing all his money? The absorbing barriers are ‘ 0 ‘ and ‘ $s$ ‘. When barrier ‘ 0 ‘ is reached the gambler is ruined.

Solution Let $p(x)$ be the probability of the particular player losing all his money if he now has $x$ units. Then we have the difference equation
$$
\begin{aligned}
&p(x)=p \cdot p(x+1)+q \cdot p(x-1) \text { if } 1<x<s-1 \
&p(1)=p \cdot p(2)=q, p(s-1)=\dot{q} \cdot p(s-2)
\end{aligned}
$$
Boundary conditions are: $p(0)=1, p(s)=0$
Auxiliary equation is $p x^{2}-x+q=0$ or $(x-1)(x-q / p)=0$
Solutions are $x=1$ and $q / p$.
General solution is $p(x)=A+B(q / p)^{x}$
From the boundary conditions $1=p(0)=A+B$
Hence
$$
\begin{aligned}
&0=p(s)=A+B(q / p)^{s} \
&B=\frac{1}{1-(q / p)^{s}}, A=\frac{-(q / p)^{s}}{1-(q / p)^{s}}
\end{aligned}
$$
The last expression follows from the fact that if $\frac{q}{p}=r \rightarrow 1$, then $\lim p(x)=1-\frac{x}{\varepsilon}$ (by L’Hospital’s Rule).

数学代写|随机过程统计代写Stochastic process statistics代考|MTH 3016

随机过程统计代考

数学代写|随机过程统计代写Stochastic process statistics代考|Different Types of Random Walks

(a) 无限制随机游走
在此,转移矩阵的元素由下式给出p一世,一世+1=p,p一世,一世−1=q, 对于所有整数一世(…,−1,0,1,2,…).
如果0<p<1,链是不可约的。然后我们有

p一世j(n)=磷(小号n=j−一世)=(n (n−j+一世)一世2)pn+j−一世2qn−j+一世2 如果 n 甚至 
=0如果n很奇怪。

p00(n)=(n n2)(pq)n/2
链的周期为 2 。
如果是短暂的p≠12和 null 经常性 ifp=12.

(b) 带有吸收障碍的随机游走
在这个游走中,转移矩阵的元素由下式给出p一世,一世+1=p,p一世,一世−1=q, (p+q=1),p00=1对所有人一世≥1.
“0”是吸收状态,其余状态都是瞬态的。0,−1,−2,−3,…凝聚成单一的吸收态‘0’。
让F一世0(n)=访问“0”的概率来自一世, 第一次在n脚步

=(一世 n)(n (n−1)/2)p(n−一世)/2q(n+一世)/2
访问“0”的概率来自一世曾经,

F一世0=∑nF一世0(n) 满足差分方程  F一世0=pF一世+1,0+qF一世−1,0 为了 一世>1,F10=pF20+q.
因此求解我们得到
$$
f_{i 0}=\left{

1 如果 p≤q (q/p)一世 如果 p≥q\正确的。

(C)R一个nd○米在一个lķ在一世吨H吨在○一个bs○rb一世nG乙一个rr一世和sH和r和吨H和和l和米和n吨s○F吨r一个ns一世吨一世○n米一个吨r一世X一世sG一世在和nb是
p_{i, i+1}=p, p_{i, i-1}=q \text { for } 1 \leq i \leq a-1, p_{00}=1, p_{aa}=1 。
$$
′0′ 和 ‘一个’是吸收和剩余的状态是短暂的。

数学代写|随机过程统计代写Stochastic process statistics代考|Examples of Random Walks with Absorbing Barriers

赌徒的终极毁灭问题赌徒
的财富形成一个具有转移矩阵
$p_{ij}=\left{的 MC

p 如果 j=一世+1 q 如果 j=一世−1 0, 否则 \正确的。一个ndi=2,3, \ldots, sp_{ij}=\左{

1 如果 j=1 0 如果 j≠1\四\右。一个nd我=1一个nd小号米○r和和Xpl一世C一世吨l是吨H和吨r一个ns一世吨一世○n米一个吨r一世X一世sG一世在和nb是磷=[1000 q0p00 0q0p0 ⋯⋯⋯⋯⋯ 00001]$

数学代写|随机过程统计代写Stochastic process statistics代考|Random Walks

让\left{X_{n}, n=0,1,2,3, \ldots\right}\left{X_{n}, n=0,1,2,3, \ldots\right}是一系列独立离散随机变量,仅取整数值,并且小号n=X1+X2…+Xn(n=0,1,2,…). 然后是序列\left{S_{n}\right}\left{S_{n}\right}是一个 MC,其转移概率由下式给出,

(米)p一世j=磷(小号米+1=j∣小号米=一世)=磷(X米+1=j−一世),一世,j=…,−2,−1,0,1,2,…
(非均匀随机游走)。
链表示粒子沿直线的随机游走,时间“跳跃”的幅度n由随机变量给出Xn. 如果X0is 表示粒子的初始位置,然后是它之后的位置n跳跃(有时n) 是(谁)给的小号n. 什么时候Xn的也是相同分布的,(n)p一世j=pj−一世在哪里pj =磷(Xn=j)。然后我们有一个均匀的随机游走(RW)。这种随机游走出现在波动理论中(离散或连续随机变量的总和)。在经典 RW 中,磷(Xn=+1)=p,p(Xn=−1)=q=1−p.

就赌博而言,这可以描述如下:
如果两个赌徒玩一系列游戏,其中特定玩家获胜的概率为p每场比赛(q=1−p是输掉比赛的概率)。如果玩家输了,他给对手一个单位的钱,如果他赢了,他从对手那里得到一个单位。如果这个特定的玩家以X金钱单位和他的对手一起s−X单位,玩家输掉所有钱的概率是多少?吸收障碍是’0’和’s’。当到达障碍“0”时,赌徒就被毁了。

解决方案让p(X)是特定玩家输掉所有钱的概率,如果他现在有X单位。然后我们有差分方程

p(X)=p⋅p(X+1)+q⋅p(X−1) 如果 1<X<s−1 p(1)=p⋅p(2)=q,p(s−1)=q˙⋅p(s−2)
边界条件为:p(0)=1,p(s)=0
辅助方程为pX2−X+q=0或者(X−1)(X−q/p)=0
解决方案是X=1和q/p.
一般解决方案是p(X)=一个+乙(q/p)X
从边界条件1=p(0)=一个+乙
因此

0=p(s)=一个+乙(q/p)s 乙=11−(q/p)s,一个=−(q/p)s1−(q/p)s
最后一个表达式来自以下事实:如果qp=r→1, 然后林p(X)=1−Xe(根据 L’Hospital 的规则)。

数学代写|随机过程统计代写Stochastic process statistics代考 请认准statistics-lab™

统计代写请认准statistics-lab™. statistics-lab™为您的留学生涯保驾护航。

金融工程代写

金融工程是使用数学技术来解决金融问题。金融工程使用计算机科学、统计学、经济学和应用数学领域的工具和知识来解决当前的金融问题,以及设计新的和创新的金融产品。

非参数统计代写

非参数统计指的是一种统计方法,其中不假设数据来自于由少数参数决定的规定模型;这种模型的例子包括正态分布模型和线性回归模型。

广义线性模型代考

广义线性模型(GLM)归属统计学领域,是一种应用灵活的线性回归模型。该模型允许因变量的偏差分布有除了正态分布之外的其它分布。

术语 广义线性模型(GLM)通常是指给定连续和/或分类预测因素的连续响应变量的常规线性回归模型。它包括多元线性回归,以及方差分析和方差分析(仅含固定效应)。

有限元方法代写

有限元方法(FEM)是一种流行的方法,用于数值解决工程和数学建模中出现的微分方程。典型的问题领域包括结构分析、传热、流体流动、质量运输和电磁势等传统领域。

有限元是一种通用的数值方法,用于解决两个或三个空间变量的偏微分方程(即一些边界值问题)。为了解决一个问题,有限元将一个大系统细分为更小、更简单的部分,称为有限元。这是通过在空间维度上的特定空间离散化来实现的,它是通过构建对象的网格来实现的:用于求解的数值域,它有有限数量的点。边界值问题的有限元方法表述最终导致一个代数方程组。该方法在域上对未知函数进行逼近。[1] 然后将模拟这些有限元的简单方程组合成一个更大的方程系统,以模拟整个问题。然后,有限元通过变化微积分使相关的误差函数最小化来逼近一个解决方案。

tatistics-lab作为专业的留学生服务机构,多年来已为美国、英国、加拿大、澳洲等留学热门地的学生提供专业的学术服务,包括但不限于Essay代写,Assignment代写,Dissertation代写,Report代写,小组作业代写,Proposal代写,Paper代写,Presentation代写,计算机作业代写,论文修改和润色,网课代做,exam代考等等。写作范围涵盖高中,本科,研究生等海外留学全阶段,辐射金融,经济学,会计学,审计学,管理学等全球99%专业科目。写作团队既有专业英语母语作者,也有海外名校硕博留学生,每位写作老师都拥有过硬的语言能力,专业的学科背景和学术写作经验。我们承诺100%原创,100%专业,100%准时,100%满意。

随机分析代写


随机微积分是数学的一个分支,对随机过程进行操作。它允许为随机过程的积分定义一个关于随机过程的一致的积分理论。这个领域是由日本数学家伊藤清在第二次世界大战期间创建并开始的。

时间序列分析代写

随机过程,是依赖于参数的一组随机变量的全体,参数通常是时间。 随机变量是随机现象的数量表现,其时间序列是一组按照时间发生先后顺序进行排列的数据点序列。通常一组时间序列的时间间隔为一恒定值(如1秒,5分钟,12小时,7天,1年),因此时间序列可以作为离散时间数据进行分析处理。研究时间序列数据的意义在于现实中,往往需要研究某个事物其随时间发展变化的规律。这就需要通过研究该事物过去发展的历史记录,以得到其自身发展的规律。

回归分析代写

多元回归分析渐进(Multiple Regression Analysis Asymptotics)属于计量经济学领域,主要是一种数学上的统计分析方法,可以分析复杂情况下各影响因素的数学关系,在自然科学、社会和经济学等多个领域内应用广泛。

MATLAB代写

MATLAB 是一种用于技术计算的高性能语言。它将计算、可视化和编程集成在一个易于使用的环境中,其中问题和解决方案以熟悉的数学符号表示。典型用途包括:数学和计算算法开发建模、仿真和原型制作数据分析、探索和可视化科学和工程图形应用程序开发,包括图形用户界面构建MATLAB 是一个交互式系统,其基本数据元素是一个不需要维度的数组。这使您可以解决许多技术计算问题,尤其是那些具有矩阵和向量公式的问题,而只需用 C 或 Fortran 等标量非交互式语言编写程序所需的时间的一小部分。MATLAB 名称代表矩阵实验室。MATLAB 最初的编写目的是提供对由 LINPACK 和 EISPACK 项目开发的矩阵软件的轻松访问,这两个项目共同代表了矩阵计算软件的最新技术。MATLAB 经过多年的发展,得到了许多用户的投入。在大学环境中,它是数学、工程和科学入门和高级课程的标准教学工具。在工业领域,MATLAB 是高效研究、开发和分析的首选工具。MATLAB 具有一系列称为工具箱的特定于应用程序的解决方案。对于大多数 MATLAB 用户来说非常重要,工具箱允许您学习应用专业技术。工具箱是 MATLAB 函数(M 文件)的综合集合,可扩展 MATLAB 环境以解决特定类别的问题。可用工具箱的领域包括信号处理、控制系统、神经网络、模糊逻辑、小波、仿真等。

R语言代写问卷设计与分析代写
PYTHON代写回归分析与线性模型代写
MATLAB代写方差分析与试验设计代写
STATA代写机器学习/统计学习代写
SPSS代写计量经济学代写
EVIEWS代写时间序列分析代写
EXCEL代写深度学习代写
SQL代写各种数据建模与可视化代写

数学代写|随机过程统计代写Stochastic process statistics代考|MXB334

如果你也在 怎样代写随机过程统计Stochastic process statistics这个学科遇到相关的难题,请随时右上角联系我们的24/7代写客服。

随机过程 用于表示在时间上发展的统计现象以及在处理这些现象时出现的理论模型,由于这些现象在许多领域都会遇到,因此这篇文章具有广泛的实际意义。

statistics-lab™ 为您的留学生涯保驾护航 在代写随机过程统计Stochastic process statistics方面已经树立了自己的口碑, 保证靠谱, 高质且原创的统计Statistics代写服务。我们的专家在代写随机过程统计Stochastic process statistics代写方面经验极为丰富,各种代写随机过程统计Stochastic process statistics相关的作业也就用不着说。

我们提供的随机过程统计Stochastic process statistics及其相关学科的代写,服务范围广, 其中包括但不限于:

  • Statistical Inference 统计推断
  • Statistical Computing 统计计算
  • Advanced Probability Theory 高等概率论
  • Advanced Mathematical Statistics 高等数理统计学
  • (Generalized) Linear Models 广义线性模型
  • Statistical Machine Learning 统计机器学习
  • Longitudinal Data Analysis 纵向数据分析
  • Foundations of Data Science 数据科学基础
数学代写|随机过程统计代写Stochastic process statistics代考|MXB334

数学代写|随机过程统计代写Stochastic process statistics代考|Theorems Regarding Finite Markov Chain

Theorem 2(a). In a M.C. with a finite number of states, there is no null state and not all states can be transient.

Proof Suppose the chain has $N<\infty$ states. If all states are transient, then letting $n \rightarrow \infty$ in the relation $\sum_{j=0}^{N} p_{i j}^{(n)}=1$ we get $0=1$ (since by Theorem $2.8$, $\lim {n \rightarrow \infty} p{i j}^{(n)}=0$ for each $j$ ), which is absured and hence not all states in a finite M.C. are transient. Consider the subchain $C_{1}$ formed by a closed set of null recurrent states. Then $\sum_{j \in C_{1}} p_{i j}^{(n)}=\alpha$ (say) $>0$. Letting $n \rightarrow \infty, 0=\alpha>0$ which is also absurd. So there cannot be any null recurrent state in a finite M.C.
Theorem 2(b). An irreducible M.C. having a finite number of states is positive recurrent.

Proof By previous theorem, there is no null recurrent state and not all states are transient. Suppose there is one transient state. Then all states are transient by Solidarity Theorem. Hence, all states are positive recurrent.

Exercise 2.6 If a finite M.C. is irreducible, aperiodic and has doubly stochastic transition matrix, then show that $\lim {n \rightarrow \infty} p{i j}^{(n)}=1 / k$, where $k$ is the number of states in the chain.

Solution If $j$ is a positive recurrent state in an aperiodic irreducible chain then
$$
p_{i j}^{(n)} \rightarrow \pi_{j}>0(\text { by Theorem 2.9) }
$$
Hence $1=\sum_{i=1}^{k} p_{i j}^{(n)}$ for all $j$ and $n \geq 1$,
$$
\begin{array}{cc}
\left(\begin{array}{cc}
p_{11} & p_{12} \ldots p_{1 k} \
p_{21} & p_{22} \ldots p_{2 k} \
\ldots & \
p_{k 1} & p_{k 2} \ldots p_{k k}
\end{array}\right)=1 \
1 & 1 \ldots 1
\end{array}
$$
Therefore $k \pi_{j}=1 \Rightarrow \pi_{j}=\frac{1}{k}$.

数学代写|随机过程统计代写Stochastic process statistics代考|Methods of Evaluation of the n-Step Transition Probability

(a) Method of Spectral Decomposition
Let $P$ be a NXN matrix with latent roots $\lambda_{1}, \ldots, \lambda_{N}$ all distinct and simple. Then $\left(P-\lambda_{j} I\right) U_{j}=0$ for the column latent vector $U_{j}$ and
$V_{i}^{\prime}\left(P-\lambda_{i} I\right)=0$ for the row latent vector $V_{i}$.
$A_{i}=U_{i} V_{i}^{\prime}$ are called latent or spectral matrix associated with $\lambda_{i}, i=1, \ldots, N$.
The following properties of $A_{i}$ ‘s are well known:

(i) $A_{i}$ ‘s are idempotent, i.e. $A_{i}^{2}=A_{i}$,
(ii) they are orthogonal, i.e. $A_{i} A_{j}=0(i \neq j)$,
(iii) they give spectral decomposition $P=\sum_{i=1}^{N} \lambda_{i} A_{i}$. It follows from (i) to (iii), that
$$
P^{k}=\left(\sum_{i=1}^{N} \lambda_{i} A_{i}\right)^{k}=\sum_{i=1}^{N} \lambda_{i}^{k} A_{i}=\sum_{i=1}^{N} \lambda_{i}^{k} U_{i} V_{i}^{\prime} .
$$
Also we know that $P^{k}=U D^{k} U^{-1}$ (by Diagonalisation Theorem) where
$$
\begin{aligned}
&U=\left(U_{1}, U_{2}, \ldots, U_{N}\right) \
&D=\left[\begin{array}{ccc}
\lambda_{1} & 0 \ldots & 0 \
0 & \lambda_{2} & \vdots \
0 & \cdots & \lambda_{N}
\end{array}\right]
\end{aligned}
$$
Since the latent vectors are determined uniquely only upto a multiplicative constant, we have chosen them such that $U_{i}^{\prime} V_{i}=1$. From $(2.21)$ one can get any power of $P$ knowing $\lambda_{i}$ ‘s and $A_{i}{ }^{\circ}$ ‘s.

Example $2.7$ We shall illustrate the last method with the help of Exercise $2.8$ of Section 2.7.

In our problem, $P=\left[\begin{array}{ccc}1 & 0 & 0 \ \frac{1}{4} & \frac{1}{2} & \frac{1}{4} \ \frac{1}{18} & \frac{8}{18} & \frac{9}{18}\end{array}\right]$ with characteristic equation
$$
\left[\begin{array}{ccc}
1-\lambda & 0 & 0 \
\frac{1}{4} & \frac{1}{2} \lambda & \frac{1}{4} \
\frac{1}{18} & \frac{8}{18} & \frac{9}{18} \lambda
\end{array}\right]=0 \text { or }(1-\lambda)\left(\lambda^{2}-\lambda+\frac{5}{36}\right)=0
$$
So the eigenvalues are $\lambda_{1}=1, \lambda_{2}=\frac{1}{6}, \lambda_{3}=\frac{5}{6}$.

数学代写|随机过程统计代写Stochastic process statistics代考|Random Walks

Let $\left{X_{n}, n=0,1,2,3, \ldots\right}$ be a sequence of independent discrete random variables taking integral values only and $S_{n}=X_{1}+X_{2} \ldots+X_{n}(n=0,1,2, \ldots)$. Then the sequence $\left{S_{n}\right}$ is a M.C. whose transition probabilities are given by,
$$
{ }^{(m)} p_{i j}=P\left(S_{m+1}=j \mid S_{m}=i\right)=P\left(X_{m+1}=j-i\right), i, j=\ldots,-2,-1,0,1,2, \ldots
$$
(non-homogeneous random walk).
The chain represents a Random walk of a particle along a straight line, the magnitude of ‘jump’ at time $n$ being given by the random variable $X_{n}$. If $X_{0}$ is denotes the initial position of a particle then its position after $n$ jumps (at time $n$ ) is given by $S_{n}$. When $X_{n}$ ‘s are also indentically distributed, ${ }^{(n)} p_{i j}=p_{j-i}$ where $p_{j}$ $=P\left(X_{n}=j\right.$ ). We have then a homogeneous Random walk (RW). Such Random walks occur in fluctuation theory (sums of discrete or continuous random variables). In classical RW, $P\left(X_{n}=+1\right)=p, p\left(X_{n}=-1\right)=q=1-p$.

数学代写|随机过程统计代写Stochastic process statistics代考|MXB334

随机过程统计代考

数学代写|随机过程统计代写Stochastic process statistics代考|Theorems Regarding Finite Markov Chain

定理 2(a)。在具有有限数量状态的 MC 中,没有零状态,并且并非所有状态都可以是瞬态的。

证明假设链有ñ<∞状态。如果所有状态都是瞬态的,那么让n→∞在关系中∑j=0ñp一世j(n)=1我们得到0=1(由于定理2.8, 林n→∞p一世j(n)=0对于每个j),这是不可靠的,因此并非有限 MC 中的所有状态都是瞬态的。考虑子链C1由一组封闭的零循环状态组成。然后∑j∈C1p一世j(n)=一个(说)>0. 让n→∞,0=一个>0这也是荒谬的。
所以在有限 MC定理 2(b)中不可能有任何零循环状态。具有有限个状态的不可约 MC 是正循环的。

证明 根据前面的定理,不存在零循环状态,并且并非所有状态都是瞬态的。假设存在一种瞬态。然后根据团结定理,所有状态都是瞬态的。因此,所有状态都是正循环的。

练习 2.6 如果一个有限 MC 是不可约的、非周期性的并且具有双重随机转移矩阵,那么证明林n→∞p一世j(n)=1/ķ, 在哪里ķ是链中的状态数。

解决方案 如果j是非周期不可约链中的正循环状态,则

p一世j(n)→圆周率j>0( 由定理 2.9) 
因此1=∑一世=1ķp一世j(n)对所有人j和n≥1,

(p11p12…p1ķ p21p22…p2ķ … pķ1pķ2…pķķ)=1 11…1
所以ķ圆周率j=1⇒圆周率j=1ķ.

数学代写|随机过程统计代写Stochastic process statistics代考|Methods of Evaluation of the n-Step Transition Probability

(a) 谱分解
法磷是一个具有潜在根的 NXN 矩阵λ1,…,λñ一切都清晰而简单。然后(磷−λj我)在j=0对于列潜在向量在j和
在一世′(磷−λ一世我)=0对于行潜在向量在一世.
一个一世=在一世在一世′被称为潜在矩阵或谱矩阵λ一世,一世=1,…,ñ.
以下属性一个一世是众所周知的:

(一世)一个一世是幂等的,即一个一世2=一个一世,
(ii) 它们是正交的,即一个一世一个j=0(一世≠j),
(iii) 他们给出谱分解磷=∑一世=1ñλ一世一个一世. 从 (i) 到 (iii) 得出,

磷ķ=(∑一世=1ñλ一世一个一世)ķ=∑一世=1ñλ一世ķ一个一世=∑一世=1ñλ一世ķ在一世在一世′.
我们也知道磷ķ=在Dķ在−1(通过对角化定理)其中

在=(在1,在2,…,在ñ) D=[λ10…0 0λ2⋮ 0⋯λñ]
由于潜在向量仅由乘法常数唯一确定,因此我们选择它们使得在一世′在一世=1. 从(2.21)可以得到任何力量磷会心λ一世’沙一个一世∘的。

例子2.7我们将在练习的帮助下说明最后一种方法2.8第 2.7 节。

在我们的问题中,磷=[100 141214 118818918]有特征方程

[1−λ00 1412λ14 118818918λ]=0 或者 (1−λ)(λ2−λ+536)=0
所以特征值为λ1=1,λ2=16,λ3=56.

数学代写|随机过程统计代写Stochastic process statistics代考|Random Walks

让\left{X_{n}, n=0,1,2,3, \ldots\right}\left{X_{n}, n=0,1,2,3, \ldots\right}是一系列独立离散随机变量,仅取整数值,并且小号n=X1+X2…+Xn(n=0,1,2,…). 然后是序列\left{S_{n}\right}\left{S_{n}\right}是一个 MC,其转移概率由下式给出,

(米)p一世j=磷(小号米+1=j∣小号米=一世)=磷(X米+1=j−一世),一世,j=…,−2,−1,0,1,2,…
(非均匀随机游走)。
链表示粒子沿直线的随机游走,时间“跳跃”的幅度n由随机变量给出Xn. 如果X0is 表示粒子的初始位置,然后是它之后的位置n跳跃(有时n) 是(谁)给的小号n. 什么时候Xn的也是相同分布的,(n)p一世j=pj−一世在哪里pj =磷(Xn=j)。然后我们有一个均匀的随机游走(RW)。这种随机游走出现在波动理论中(离散或连续随机变量的总和)。在经典 RW 中,磷(Xn=+1)=p,p(Xn=−1)=q=1−p.

数学代写|随机过程统计代写Stochastic process statistics代考 请认准statistics-lab™

统计代写请认准statistics-lab™. statistics-lab™为您的留学生涯保驾护航。

金融工程代写

金融工程是使用数学技术来解决金融问题。金融工程使用计算机科学、统计学、经济学和应用数学领域的工具和知识来解决当前的金融问题,以及设计新的和创新的金融产品。

非参数统计代写

非参数统计指的是一种统计方法,其中不假设数据来自于由少数参数决定的规定模型;这种模型的例子包括正态分布模型和线性回归模型。

广义线性模型代考

广义线性模型(GLM)归属统计学领域,是一种应用灵活的线性回归模型。该模型允许因变量的偏差分布有除了正态分布之外的其它分布。

术语 广义线性模型(GLM)通常是指给定连续和/或分类预测因素的连续响应变量的常规线性回归模型。它包括多元线性回归,以及方差分析和方差分析(仅含固定效应)。

有限元方法代写

有限元方法(FEM)是一种流行的方法,用于数值解决工程和数学建模中出现的微分方程。典型的问题领域包括结构分析、传热、流体流动、质量运输和电磁势等传统领域。

有限元是一种通用的数值方法,用于解决两个或三个空间变量的偏微分方程(即一些边界值问题)。为了解决一个问题,有限元将一个大系统细分为更小、更简单的部分,称为有限元。这是通过在空间维度上的特定空间离散化来实现的,它是通过构建对象的网格来实现的:用于求解的数值域,它有有限数量的点。边界值问题的有限元方法表述最终导致一个代数方程组。该方法在域上对未知函数进行逼近。[1] 然后将模拟这些有限元的简单方程组合成一个更大的方程系统,以模拟整个问题。然后,有限元通过变化微积分使相关的误差函数最小化来逼近一个解决方案。

tatistics-lab作为专业的留学生服务机构,多年来已为美国、英国、加拿大、澳洲等留学热门地的学生提供专业的学术服务,包括但不限于Essay代写,Assignment代写,Dissertation代写,Report代写,小组作业代写,Proposal代写,Paper代写,Presentation代写,计算机作业代写,论文修改和润色,网课代做,exam代考等等。写作范围涵盖高中,本科,研究生等海外留学全阶段,辐射金融,经济学,会计学,审计学,管理学等全球99%专业科目。写作团队既有专业英语母语作者,也有海外名校硕博留学生,每位写作老师都拥有过硬的语言能力,专业的学科背景和学术写作经验。我们承诺100%原创,100%专业,100%准时,100%满意。

随机分析代写


随机微积分是数学的一个分支,对随机过程进行操作。它允许为随机过程的积分定义一个关于随机过程的一致的积分理论。这个领域是由日本数学家伊藤清在第二次世界大战期间创建并开始的。

时间序列分析代写

随机过程,是依赖于参数的一组随机变量的全体,参数通常是时间。 随机变量是随机现象的数量表现,其时间序列是一组按照时间发生先后顺序进行排列的数据点序列。通常一组时间序列的时间间隔为一恒定值(如1秒,5分钟,12小时,7天,1年),因此时间序列可以作为离散时间数据进行分析处理。研究时间序列数据的意义在于现实中,往往需要研究某个事物其随时间发展变化的规律。这就需要通过研究该事物过去发展的历史记录,以得到其自身发展的规律。

回归分析代写

多元回归分析渐进(Multiple Regression Analysis Asymptotics)属于计量经济学领域,主要是一种数学上的统计分析方法,可以分析复杂情况下各影响因素的数学关系,在自然科学、社会和经济学等多个领域内应用广泛。

MATLAB代写

MATLAB 是一种用于技术计算的高性能语言。它将计算、可视化和编程集成在一个易于使用的环境中,其中问题和解决方案以熟悉的数学符号表示。典型用途包括:数学和计算算法开发建模、仿真和原型制作数据分析、探索和可视化科学和工程图形应用程序开发,包括图形用户界面构建MATLAB 是一个交互式系统,其基本数据元素是一个不需要维度的数组。这使您可以解决许多技术计算问题,尤其是那些具有矩阵和向量公式的问题,而只需用 C 或 Fortran 等标量非交互式语言编写程序所需的时间的一小部分。MATLAB 名称代表矩阵实验室。MATLAB 最初的编写目的是提供对由 LINPACK 和 EISPACK 项目开发的矩阵软件的轻松访问,这两个项目共同代表了矩阵计算软件的最新技术。MATLAB 经过多年的发展,得到了许多用户的投入。在大学环境中,它是数学、工程和科学入门和高级课程的标准教学工具。在工业领域,MATLAB 是高效研究、开发和分析的首选工具。MATLAB 具有一系列称为工具箱的特定于应用程序的解决方案。对于大多数 MATLAB 用户来说非常重要,工具箱允许您学习应用专业技术。工具箱是 MATLAB 函数(M 文件)的综合集合,可扩展 MATLAB 环境以解决特定类别的问题。可用工具箱的领域包括信号处理、控制系统、神经网络、模糊逻辑、小波、仿真等。

R语言代写问卷设计与分析代写
PYTHON代写回归分析与线性模型代写
MATLAB代写方差分析与试验设计代写
STATA代写机器学习/统计学习代写
SPSS代写计量经济学代写
EVIEWS代写时间序列分析代写
EXCEL代写深度学习代写
SQL代写各种数据建模与可视化代写

数学代写|随机过程统计代写Stochastic process statistics代考|MTH 3016

如果你也在 怎样代写随机过程统计Stochastic process statistics这个学科遇到相关的难题,请随时右上角联系我们的24/7代写客服。

随机过程 用于表示在时间上发展的统计现象以及在处理这些现象时出现的理论模型,由于这些现象在许多领域都会遇到,因此这篇文章具有广泛的实际意义。

statistics-lab™ 为您的留学生涯保驾护航 在代写随机过程统计Stochastic process statistics方面已经树立了自己的口碑, 保证靠谱, 高质且原创的统计Statistics代写服务。我们的专家在代写随机过程统计Stochastic process statistics代写方面经验极为丰富,各种代写随机过程统计Stochastic process statistics相关的作业也就用不着说。

我们提供的随机过程统计Stochastic process statistics及其相关学科的代写,服务范围广, 其中包括但不限于:

  • Statistical Inference 统计推断
  • Statistical Computing 统计计算
  • Advanced Probability Theory 高等概率论
  • Advanced Mathematical Statistics 高等数理统计学
  • (Generalized) Linear Models 广义线性模型
  • Statistical Machine Learning 统计机器学习
  • Longitudinal Data Analysis 纵向数据分析
  • Foundations of Data Science 数据科学基础
数学代写|随机过程统计代写Stochastic process statistics代考|MTH 3016

数学代写|随机过程统计代写Stochastic process statistics代考|Limit Theorems for Markov Chain

Definition $2.10$ Let $d(\mathrm{i})$ be the greatest common divisor of those $n \geq 1$ for which $p_{i i}^{(n)}>0$. Then $d(i)$ is called the period of the state $i$. If $d(i)=1$, then the state $i$ is called aperiodic.
Note $i \leftrightarrow j$, then $d(i)=d(j)$
There exists $n_{1}$ and $n_{2}$ such that $p_{i j}^{\left(n_{1}\right)}>0$ and $p_{j i}^{\left(n_{2}\right)}>0$.
Now $p_{i i}^{\left(n_{1}+n_{2}\right)} \geq p_{i j}^{\left(n_{1}\right)} p_{j i}^{\left(n_{2}\right)}>0$ and hence $d(i)$ is a divisor of $n_{1}+n_{2}$.
If $p_{j j}^{(n)}>0$, then $p_{i i}^{\left(n_{1}+n+n_{2}\right)} \geq p_{i j}^{\left(n_{1}\right)} p_{j j}^{(n)} p_{j i}^{\left(n_{2}\right)}>0$ (by Chapman Kolmogorov equation).

Hence, $d(i)$ is a divisor of $n_{1}+n+n_{2}$. So $d(i)$ must be a divisor of $n$ if $p_{j i}^{(n)}>0$

Thus $d(i)$ is a divisor of $\left{n \geq 1: p_{j j}^{(n)}>0\right}$. Since $d(j)$ is the largest of such divisors, $d(i) \leq d(j)$. Hence, by symmetry $d(j) \leq d(i)$.
Hence $d(i)=d(j)$. Therefore having a period $d$ is a class property.
Note If $p_{i i}>0$, then $d(i)=1$ and this implies that a sufficient condition for an irreducible M.C. to be aperiodic is that $p_{i i}>0$ for some $i \in S$. Hence a queueing chain is aperiodic.
Theorem $2.7$ Limit Theorem (for diagonal elements)
Let $j$ be any state in a M.C. As $n \rightarrow \infty$.
(i) if $j$ is transient, then $p_{j j}^{(n)} \rightarrow 0$
(ii) if $j$ is null recurrent, then $p_{j j}^{(n)} \rightarrow 0$
(iii) if $j$ is positive (recurrent) and
(a) aperiodic, then $p_{j j}^{(n)} \rightarrow \frac{1}{\sum_{n=1}^{\infty} n f_{j j}^{(n)}}=\frac{1}{\mu_{j}}$ (mean recurrence time of $j$ )(b) periodic with period $d(j)$ then $p_{J \prime}^{(n d(j))} \rightarrow \frac{d(j)}{\mu_{j}}$. Write $d(j) / \mu_{j}=\pi_{j}$.

数学代写|随机过程统计代写Stochastic process statistics代考|Special Chains and Foster Type Theorems

If the Markov Chain is infinite, the number of equations given by $\pi(P-I)=0$ will be infinite involving an infinite number of unknowns. In some particular cases we can solve these equations. The following examples will illustrate this point.
Example $2.5$ Birth and.Death Chain (Non-Homogeneous Random Walk) Consider a birth and death chain on ${0,1,2, \ldots, d}$ or a set of non-negative integers i.e. where $d=\infty$. Assume that the chain is irreducible i.e. $p_{j}>0$ and $q_{j}>0$ in case $0 \leq j \leq d$ (i.e. when $d$ is finite) $p_{j}>0$ for $0 \leq j<\infty$ and $q_{j}>0$ for $0<j<\infty$ if $d$ is infinite. Consider the transition matrix
$$
\left(\begin{array}{cccccc}
r_{0} & p_{0} & 0 & \cdots & \cdot & \
q_{1} & r_{1} & p_{1} & 0 & \cdots & \
0 & q_{2} & r_{2} & p_{2} & 0 & . \
0 & 0 & q_{3} & r_{3} & p_{3} & 0 \
\cdot & \cdot & \cdot & \cdot & \cdots &
\end{array}\right)
$$
when $d<\infty$ we assume that $r_{i}=0$ for $i \geq 0$ and $p_{0}=1$.
Particular Case: First consider that $d$ is still infinite and $r_{1}=0$ for $i \geq 0$, $p_{0}=1$. The stationary distribution is given by or $X=X P$. Let $x_{0} \neq 0$. Then
$$
\begin{aligned}
&x_{0}=x_{1} q_{1}, \
&x_{1}=x_{0}+x_{2} q_{2}, \
&x_{3}=x_{2} p_{2}+x_{4} q_{4}, \
&x_{4}=\ldots \
&\ldots
\end{aligned}
$$
Define
$$
y_{i}=\frac{x_{i}}{x_{0}}, y_{0}=1, i=1,2,3, \ldots
$$
Then
$$
\begin{aligned}
&y_{1}=1 / q_{1}, y_{1}=1+y_{2} q_{2} \text { or } y_{2}=\frac{y_{1}-1}{q_{2}}=\frac{1-q_{1}}{q_{1} q_{2}}=\frac{p_{1}}{q_{1} q_{2}} \
&y_{3}=\frac{p_{1} p_{2}}{q_{1} q_{2} q_{3}}, \ldots, y_{n}=\frac{p_{1} p_{2} \ldots p_{n-1}}{q_{1} q_{2} \ldots q_{n}}>0 \quad \text { for all } n=1,2, \ldots
\end{aligned}
$$
(by assumption that all $p, q$ ‘s are $>0$ ).

数学代写|随机过程统计代写Stochastic process statistics代考|Foster type theorems

The following theorems, associated with Foster, give criteria for transient and recurrent chains in terms of solution of certain equations. Assume that the M.C. is irreducible.

Theorem 2.11 (Foster, 1953) Let the Markov chain be irreducible. Assume that there exists $x_{k} . k \in S$ such that $x_{k}=\sum_{k \in S} x_{i} p_{i k}$ and $0<\sum_{k \in S}\left|x_{k}\right|<\infty$. Then the Markóv Chain is positive recurrent (this is a soort of converse of Theoremem $2.9$ ). Proof Since $y_{k}=\frac{1}{\sum_{k \in S}\left|x_{k}\right|}>0, \sum_{k \in S} y_{k}=1$.

Without loss of generality $\left{x_{k}, k \in S\right}$ is a stationary distribution of a M.C. Then

$$
x_{k}=\sum_{k \in S} x_{i} p_{i k}^{(n)} \text { for all } n=1,2, \ldots
$$
Suppose that there is no positive state.
Since the M.C. is irreducible, then all the states are either transient or null. In that case $p_{i k}^{(n)} \rightarrow 0$ as $n \rightarrow \infty$ for all $i, k \in S$. By Lebesgue Dominated Convergence Theorem, taking $n \rightarrow \infty$ in (2.19)
$$
x_{k}=\sum_{i \in S}\left(x_{i}\right), 0=0 \text { for all } k \in S
$$
But $0<\sum_{k \in S} x_{k}<\infty$ is a contradiction to $(2.20)$.
Hence, there is at least one positive recurrent state. Since M.C. is irreducible, by Solidarity Theorem the M.C. must be positive recurrent.

Conclusion An ireducible aperiodic M.C. has a stationary distribution iff all states are positive recurrent.

数学代写|随机过程统计代写Stochastic process statistics代考|MTH 3016

随机过程统计代考

数学代写|随机过程统计代写Stochastic process statistics代考|Limit Theorems for Markov Chain

定义2.10让d(一世)成为那些的最大公约数n≥1为此p一世一世(n)>0. 然后d(一世)称为状态周期一世. 如果d(一世)=1,那么状态一世称为非周期性。
笔记一世↔j, 然后d(一世)=d(j)
那里存在n1和n2这样p一世j(n1)>0和pj一世(n2)>0.
现在p一世一世(n1+n2)≥p一世j(n1)pj一世(n2)>0因此d(一世)是一个除数n1+n2.
如果pjj(n)>0, 然后p一世一世(n1+n+n2)≥p一世j(n1)pjj(n)pj一世(n2)>0(通过查普曼科尔莫哥洛夫方程)。

因此,d(一世)是一个除数n1+n+n2. 所以d(一世)必须是的除数n如果pj一世(n)>0

因此d(一世)是一个除数\left{n \geq 1: p_{j j}^{(n)}>0\right}\left{n \geq 1: p_{j j}^{(n)}>0\right}. 自从d(j)是此类除数中最大的,d(一世)≤d(j). 因此,通过对称d(j)≤d(一世).
因此d(一世)=d(j). 因此有一个时期d是类属性。
注意如果p一世一世>0, 然后d(一世)=1这意味着不可约 MC 是非周期性的充分条件是p一世一世>0对于一些一世∈小号. 因此,排队链是非周期性的。
定理2.7极限定理(对角元素)
让j成为 MC As 中的任何状态n→∞.
(i) 如果j是瞬态的,那么pjj(n)→0
(ii) 如果j是零循环的,那么pjj(n)→0
(iii) 如果j是正的(经常性的)和
(a) 非周期性的,那么pjj(n)→1∑n=1∞nFjj(n)=1μj(平均复发时间j)(b) 有周期的周期性d(j)然后pĴ′(nd(j))→d(j)μj. 写d(j)/μj=圆周率j.

数学代写|随机过程统计代写Stochastic process statistics代考|Special Chains and Foster Type Theorems

如果马尔可夫链是无限的,则方程数由下式给出圆周率(磷−我)=0将是无限的,涉及无限数量的未知数。在某些特定情况下,我们可以求解这些方程。下面的例子将说明这一点。
例子2.5Birth and.Death Chain (Non-Homogeneous Random Walk) 考虑一个出生和死亡链0,1,2,…,d或一组非负整数,即其中d=∞. 假设链是不可约的,即pj>0和qj>0如果0≤j≤d(即当d是有限的)pj>0为了0≤j<∞和qj>0为了0<j<∞如果d是无限的。考虑转移矩阵

(r0p00⋯⋅ q1r1p10⋯ 0q2r2p20. 00q3r3p30 ⋅⋅⋅⋅⋯)
什么时候d<∞我们假设r一世=0为了一世≥0和p0=1.
特殊情况:首先考虑d仍然是无限的并且r1=0为了一世≥0, p0=1. 平稳分布由下式给出X=X磷. 让X0≠0. 然后

X0=X1q1, X1=X0+X2q2, X3=X2p2+X4q4, X4=… …
定义

是一世=X一世X0,是0=1,一世=1,2,3,…
然后

是1=1/q1,是1=1+是2q2 或者 是2=是1−1q2=1−q1q1q2=p1q1q2 是3=p1p2q1q2q3,…,是n=p1p2…pn−1q1q2…qn>0 对所有人 n=1,2,…
(假设所有p,q是>0 ).

数学代写|随机过程统计代写Stochastic process statistics代考|Foster type theorems

以下与 Foster 相关的定理根据某些方程的解给出了瞬态链和循环链的标准。假设 MC 是不可约的。

定理 2.11 (Foster, 1953) 设马尔可夫链不可约。假设存在Xķ.ķ∈小号这样Xķ=∑ķ∈小号X一世p一世ķ和0<∑ķ∈小号|Xķ|<∞. 那么马尔科夫链是正循环的(这是定理的一种逆2.9)。证明自是ķ=1∑ķ∈小号|Xķ|>0,∑ķ∈小号是ķ=1.

不失一般性\left{x_{k}, k \in S\right}\left{x_{k}, k \in S\right}是一个 MC 的平稳分布

Xķ=∑ķ∈小号X一世p一世ķ(n) 对所有人 n=1,2,…
假设没有积极的状态。
由于 MC 是不可约的,那么所有状态要么是瞬态的,要么是空的。在这种情况下p一世ķ(n)→0作为n→∞对所有人一世,ķ∈小号. 由勒贝格支配收敛定理,取n→∞在 (2.19)

Xķ=∑一世∈小号(X一世),0=0 对所有人 ķ∈小号
但0<∑ķ∈小号Xķ<∞是矛盾的(2.20).
因此,至少存在一种积极的复发状态。由于 MC 是不可约的,根据团结定理,MC 必须是正循环的。

结论当所有状态都为正循环时,一个不可约非周期MC具有平稳分布。

数学代写|随机过程统计代写Stochastic process statistics代考 请认准statistics-lab™

统计代写请认准statistics-lab™. statistics-lab™为您的留学生涯保驾护航。

金融工程代写

金融工程是使用数学技术来解决金融问题。金融工程使用计算机科学、统计学、经济学和应用数学领域的工具和知识来解决当前的金融问题,以及设计新的和创新的金融产品。

非参数统计代写

非参数统计指的是一种统计方法,其中不假设数据来自于由少数参数决定的规定模型;这种模型的例子包括正态分布模型和线性回归模型。

广义线性模型代考

广义线性模型(GLM)归属统计学领域,是一种应用灵活的线性回归模型。该模型允许因变量的偏差分布有除了正态分布之外的其它分布。

术语 广义线性模型(GLM)通常是指给定连续和/或分类预测因素的连续响应变量的常规线性回归模型。它包括多元线性回归,以及方差分析和方差分析(仅含固定效应)。

有限元方法代写

有限元方法(FEM)是一种流行的方法,用于数值解决工程和数学建模中出现的微分方程。典型的问题领域包括结构分析、传热、流体流动、质量运输和电磁势等传统领域。

有限元是一种通用的数值方法,用于解决两个或三个空间变量的偏微分方程(即一些边界值问题)。为了解决一个问题,有限元将一个大系统细分为更小、更简单的部分,称为有限元。这是通过在空间维度上的特定空间离散化来实现的,它是通过构建对象的网格来实现的:用于求解的数值域,它有有限数量的点。边界值问题的有限元方法表述最终导致一个代数方程组。该方法在域上对未知函数进行逼近。[1] 然后将模拟这些有限元的简单方程组合成一个更大的方程系统,以模拟整个问题。然后,有限元通过变化微积分使相关的误差函数最小化来逼近一个解决方案。

tatistics-lab作为专业的留学生服务机构,多年来已为美国、英国、加拿大、澳洲等留学热门地的学生提供专业的学术服务,包括但不限于Essay代写,Assignment代写,Dissertation代写,Report代写,小组作业代写,Proposal代写,Paper代写,Presentation代写,计算机作业代写,论文修改和润色,网课代做,exam代考等等。写作范围涵盖高中,本科,研究生等海外留学全阶段,辐射金融,经济学,会计学,审计学,管理学等全球99%专业科目。写作团队既有专业英语母语作者,也有海外名校硕博留学生,每位写作老师都拥有过硬的语言能力,专业的学科背景和学术写作经验。我们承诺100%原创,100%专业,100%准时,100%满意。

随机分析代写


随机微积分是数学的一个分支,对随机过程进行操作。它允许为随机过程的积分定义一个关于随机过程的一致的积分理论。这个领域是由日本数学家伊藤清在第二次世界大战期间创建并开始的。

时间序列分析代写

随机过程,是依赖于参数的一组随机变量的全体,参数通常是时间。 随机变量是随机现象的数量表现,其时间序列是一组按照时间发生先后顺序进行排列的数据点序列。通常一组时间序列的时间间隔为一恒定值(如1秒,5分钟,12小时,7天,1年),因此时间序列可以作为离散时间数据进行分析处理。研究时间序列数据的意义在于现实中,往往需要研究某个事物其随时间发展变化的规律。这就需要通过研究该事物过去发展的历史记录,以得到其自身发展的规律。

回归分析代写

多元回归分析渐进(Multiple Regression Analysis Asymptotics)属于计量经济学领域,主要是一种数学上的统计分析方法,可以分析复杂情况下各影响因素的数学关系,在自然科学、社会和经济学等多个领域内应用广泛。

MATLAB代写

MATLAB 是一种用于技术计算的高性能语言。它将计算、可视化和编程集成在一个易于使用的环境中,其中问题和解决方案以熟悉的数学符号表示。典型用途包括:数学和计算算法开发建模、仿真和原型制作数据分析、探索和可视化科学和工程图形应用程序开发,包括图形用户界面构建MATLAB 是一个交互式系统,其基本数据元素是一个不需要维度的数组。这使您可以解决许多技术计算问题,尤其是那些具有矩阵和向量公式的问题,而只需用 C 或 Fortran 等标量非交互式语言编写程序所需的时间的一小部分。MATLAB 名称代表矩阵实验室。MATLAB 最初的编写目的是提供对由 LINPACK 和 EISPACK 项目开发的矩阵软件的轻松访问,这两个项目共同代表了矩阵计算软件的最新技术。MATLAB 经过多年的发展,得到了许多用户的投入。在大学环境中,它是数学、工程和科学入门和高级课程的标准教学工具。在工业领域,MATLAB 是高效研究、开发和分析的首选工具。MATLAB 具有一系列称为工具箱的特定于应用程序的解决方案。对于大多数 MATLAB 用户来说非常重要,工具箱允许您学习应用专业技术。工具箱是 MATLAB 函数(M 文件)的综合集合,可扩展 MATLAB 环境以解决特定类别的问题。可用工具箱的领域包括信号处理、控制系统、神经网络、模糊逻辑、小波、仿真等。

R语言代写问卷设计与分析代写
PYTHON代写回归分析与线性模型代写
MATLAB代写方差分析与试验设计代写
STATA代写机器学习/统计学习代写
SPSS代写计量经济学代写
EVIEWS代写时间序列分析代写
EXCEL代写深度学习代写
SQL代写各种数据建模与可视化代写

数学代写|随机过程统计代写Stochastic process statistics代考|MTH7090

如果你也在 怎样代写随机过程统计Stochastic process statistics这个学科遇到相关的难题,请随时右上角联系我们的24/7代写客服。

随机过程 用于表示在时间上发展的统计现象以及在处理这些现象时出现的理论模型,由于这些现象在许多领域都会遇到,因此这篇文章具有广泛的实际意义。

statistics-lab™ 为您的留学生涯保驾护航 在代写随机过程统计Stochastic process statistics方面已经树立了自己的口碑, 保证靠谱, 高质且原创的统计Statistics代写服务。我们的专家在代写随机过程统计Stochastic process statistics代写方面经验极为丰富,各种代写随机过程统计Stochastic process statistics相关的作业也就用不着说。

我们提供的随机过程统计Stochastic process statistics及其相关学科的代写,服务范围广, 其中包括但不限于:

  • Statistical Inference 统计推断
  • Statistical Computing 统计计算
  • Advanced Probability Theory 高等概率论
  • Advanced Mathematical Statistics 高等数理统计学
  • (Generalized) Linear Models 广义线性模型
  • Statistical Machine Learning 统计机器学习
  • Longitudinal Data Analysis 纵向数据分析
  • Foundations of Data Science 数据科学基础
数学代写|随机过程统计代写Stochastic process statistics代考|MTH7090

数学代写|随机过程统计代写Stochastic process statistics代考|Classification of state

Let $f_{i j}^{(n)}=P\left[X_{n}=j, X_{n-1} \neq j, X_{n-2} \neq j, \ldots, X_{1} \neq j \mid X_{0}=i\right]$, i.e. probability of arriving at $j$ at time $n$ for the first time, given that the process starts at $i$.
Define $f_{i j}^{(0)}=0$. Note that $f_{i j}^{(n)}=P\left[T_{i j}=n\right]$, where
$$
T_{i j}=\min \left{n: X_{n}=j \mid X_{0}=i\right}
$$
$f_{i j}^{(n)}$ are called the first entrance probability at $n$th step if $i \neq j$ and recurrence probability at the $n$th step.

Note $f_{i j}^{(1)}=p_{j j}$ gives the diagonal of the transition matrix.
Theorem 2.4 $p_{i j}^{(n)}=\sum_{m=1}^{n} f_{i j}^{(m)} p_{j j}^{(n-m)}$ for all $m=1,2, \ldots n$.
Proof $p_{i j}^{(n)}=P\left[X_{n}=j \mid X_{0}=i\right]$
$$
\begin{aligned}
&\left.=\sum_{m=1}^{n} \frac{P\left[X_{n i} \equiv j\right.}{A}, \frac{X_{m}=j, X_{m-1} \neq j, \ldots, X_{1} \neq j}{B_{m}} \mid X_{0}=i\right] \
&=\sum_{m=1}^{n} P\left[A B_{m} \mid C\right]
\end{aligned}
$$
where $B_{m}$ are disjoint (mutually exclusive) and $\bigcup_{m-1}^{n} B_{m} \supset A$.
Hence
$$
\begin{aligned}
P_{i j}^{(n)} &=\sum_{m=1}^{n} \frac{P\left(A B_{m} C\right) P\left(B_{m} C\right)}{P(C) P\left(B_{m} C\right)} \
&=\sum_{m=1}^{n} P\left(A \mid B_{m} C\right) P\left(B_{m} \mid C\right) \
&=\sum_{m=1}^{n} P\left[X_{n}=j \mid X_{m}=j, X_{m-1} \neq j, \ldots, X_{1} \neq j, X_{0}=i\right] \
P\left[X_{m}\right.&\left.=j, X_{m-1} \neq j, \ldots, X_{1} \neq j \mid X_{0}=i\right] \
&=\sum_{m=1}^{n} P\left(X_{n}=j \mid X_{m}=j\right) f_{i j}^{(m)} \
&=\sum_{m=1}^{n} P_{j i}^{(n-m)} f_{i j}^{(m)}
\end{aligned}
$$

数学代写|随机过程统计代写Stochastic process statistics代考|A few important theorem

Let $\left{a_{n}\right}$ be a sequence of real numbers such that $0 \leq a_{n} \leq 1, n=0,1,2, \ldots$.
Let $A(z)=\sum_{n=0}^{\infty} a_{n} z^{n}$ be the generating function of $\left{a_{n}\right}$

  1. Pringsheim : $A(z)$ converges in a circle $|z|<r$, where $r \leq 1 . Z=r$ is a singularity of $A(z)$.
  2. Abel : If $\sum_{n=0}^{\infty} a_{n}=a<\infty$, then $\lim _{: \rightarrow 1-} A(Z)=a$.
  3. Tauber: If $\lim {z \rightarrow 1-} A(Z)=a \leq \infty$, then $\sum{n=0}^{\infty} a_{n}=a$.
  4. Cesaro-Abel: If $\lim {n \rightarrow \infty} \frac{1}{n} \sum{k=1}^{\infty} a_{k}=L<\infty$, then
    $$
    \lim {z \rightarrow 1{-}}(1-z) A(z)=L .
    $$
  5. Cesaro-Tauber: If $\lim _{z \rightarrow 1-}(1-z) A(z)=L<\infty$, then
  6. $$
  7. \lim {z \rightarrow 1–} \frac{1}{n} \sum{k=0}^{n} a_{k}=L .
  8. $$
  9. For proofs of 2 and 3 see the book of Karlin (page 46 , Introduction of Stochastic process) and proofs of 1,4 and 5 see the book of Tichmarsh-Theory of Functions.
  10. Let $n=0,1,2, \ldots$
  1. (Lebesgue) Dominated Convergence Theorem
    If (i) $\lim {n \rightarrow \infty} a{n, m}$ exists for every $m$
    (ii) $\left|a_{n m}\right| \leq b_{m}$ (independent of $n$ ) for all $m \geq 0$
    (iii) $\sum_{m=0}^{\infty} b_{m}<\infty$
    then $\lim {n \rightarrow \infty} \sum{m=0}^{\infty} a_{n m}=\sum_{m=0}^{\infty} \lim {n \rightarrow \infty} b{n m}$.
  2. Fatou’s Lemma
    If (i) $a_{n m} \geq 0$ for all $m, n$,
    (ii) $\lim {n \rightarrow \infty} a{n m}$ exists for all $m$,
    then $\quad \lim {n \rightarrow \infty}\left[\sum{m=0}^{\infty} a_{n m}\right] \geq \sum_{m=0}^{\infty}\left[\lim {n \rightarrow \infty} a{n m}\right]$.
  3. Fubini’s Theorem
    In order that
    $$
    \sum_{n=0}^{\infty} \sum_{m=0}^{\infty} a_{n m}=\sum_{m=0}^{\infty} \sum_{n=0}^{\infty} a_{n m}
    $$
    it is sufficient that at least one of the following conditions is satisfied:
    (i) $a_{n m} \geq 0$ for all $n . m$
    (ii) $\sum_{n=0}^{\infty} \sum_{m=0}^{\infty}\left|a_{n m}\right|<\infty$
    (iii) $\sum_{m=0}^{\infty} \sum_{n=0}^{\infty}\left|a_{n m}\right|<\infty$

数学代写|随机过程统计代写Stochastic process statistics代考|Homogeneous Random Walk

Here state space is given by
$$
\begin{aligned}
&S={\ldots,-2,-1,0,1,2 \ldots} \
&p_{i}=p, q_{i}=q \text { for all } i \geq 1 .
\end{aligned}
$$
This is an irreducible M.C. Hence by solidarity theorem it is enough to consider the state ${0}$ only. The $n$-step recurrence probability is ${0}$ is transient iff $\sum_{n=0}^{\infty} P_{00}^{(n)}<\infty$ and recurrent iff $\sum_{n=0}^{\infty} P_{00}^{(n)}=\infty$ (by Theorem 2.5) $$ \begin{aligned} \sum_{m=1}^{\infty}\left(\begin{array}{c} 2 m \ m \end{array}\right) p^{m} q^{m} & \cong \sum_{m=1}^{\infty} \frac{(4 p q)^{m}}{(\pi m)^{1 / 2}}<\infty \text { if } 4 p q<1 \\ &=\infty \text { if } 4 p q \geq 1 \end{aligned} $$ (using Stirling’s approximation for $m$ ! $\cong \sqrt{2 \pi} e^{-m} m^{m+1 / 2}$ ) $4 p q>1$ is impossible for if $4 p q>(p+q)^{2}$ then $0>(p-q)^{2}$.
Hence
$4 p q<1$ if $p \neq q$
$$
=1 \text { if } p=q=\frac{1}{2} \text {. }
$$
Therefore $\sum_{m=0}^{\infty} p_{00}^{(2 m)}$ converges faster than the geometric series $\sum_{m}(4 p q)^{m}$ if $p \neq 1 / 2$.
Hence Random walk is recurrent iff $p=\frac{1}{2}$ and transient iff $p \neq \frac{1}{2}$.
We have shown in Exercise $2.1$ that a symmetric Random walk in one dimension is recurrent. Similarly it can be proved that in 2-dimensions a symmetric Random walk is recurrent. But Polya proved that in $k \geq 3$ dimensions a symmetric Random walk is transient.

数学代写|随机过程统计代写Stochastic process statistics代考|MTH7090

随机过程统计代考

数学代写|随机过程统计代写Stochastic process statistics代考|Classification of state

让F一世j(n)=磷[Xn=j,Xn−1≠j,Xn−2≠j,…,X1≠j∣X0=一世],即到达的概率j有时n第一次,鉴于该过程开始于一世.
定义F一世j(0)=0. 注意F一世j(n)=磷[吨一世j=n], 在哪里

T_{i j}=\min \left{n: X_{n}=j \mid X_{0}=i\right}T_{i j}=\min \left{n: X_{n}=j \mid X_{0}=i\right}
F一世j(n)被称为第一次进入概率n如果一世≠j和复发概率n第一步。

笔记F一世j(1)=pjj给出转移矩阵的对角线.
定理 2.4p一世j(n)=∑米=1nF一世j(米)pjj(n−米)对所有人米=1,2,…n.
证明p一世j(n)=磷[Xn=j∣X0=一世]

=∑米=1n磷[Xn一世≡j一个,X米=j,X米−1≠j,…,X1≠j乙米∣X0=一世] =∑米=1n磷[一个乙米∣C]
在哪里乙米是不相交的(互斥的)并且⋃米−1n乙米⊃一个.
因此

磷一世j(n)=∑米=1n磷(一个乙米C)磷(乙米C)磷(C)磷(乙米C) =∑米=1n磷(一个∣乙米C)磷(乙米∣C) =∑米=1n磷[Xn=j∣X米=j,X米−1≠j,…,X1≠j,X0=一世] 磷[X米=j,X米−1≠j,…,X1≠j∣X0=一世] =∑米=1n磷(Xn=j∣X米=j)F一世j(米) =∑米=1n磷j一世(n−米)F一世j(米)

数学代写|随机过程统计代写Stochastic process statistics代考|A few important theorem

让\left{a_{n}\right}\left{a_{n}\right}是一个实数序列,使得0≤一个n≤1,n=0,1,2,….
让一个(和)=∑n=0∞一个n和n是的生成函数\left{a_{n}\right}\left{a_{n}\right}

  1. 普林斯海姆:一个(和)汇聚成一个圆圈|和|<r, 在哪里r≤1.从=r是一个奇点一个(和).
  2. 阿贝尔:如果∑n=0∞一个n=一个<∞, 然后林:→1−一个(从)=一个.
  3. 陶伯:如果林和→1−一个(从)=一个≤∞, 然后∑n=0∞一个n=一个.
  4. 塞萨罗-阿贝尔:如果林n→∞1n∑ķ=1∞一个ķ=大号<∞, 然后
    林和→1−(1−和)一个(和)=大号.
  5. 塞萨罗-陶伯:如果林和→1−(1−和)一个(和)=大号<∞, 然后
  6. $$
  7. \lim {z \rightarrow 1–} \frac{1}{n} \sum{k=0}^{n} a_{k}=L 。
  8. $$
  9. 对于 2 和 3 的证明,请参阅 Karlin 的书(第 46 页,随机过程的介绍)和 1,4 和 5 的证明,请参阅 Tichmarsh-Theory of Functions 的书。
  10. 让n=0,1,2,…
  1. (Lebesgue) 支配收敛定理
    If (i)林n→∞一个n,米存在于每个米
    (二)|一个n米|≤b米(独立于n) 对所有人米≥0
    ㈢∑米=0∞b米<∞
    然后林n→∞∑米=0∞一个n米=∑米=0∞林n→∞bn米.
  2. Fatou 引理
    If (i)一个n米≥0对所有人米,n,
    (ii)林n→∞一个n米为所有人而存在米,
    那么林n→∞[∑米=0∞一个n米]≥∑米=0∞[林n→∞一个n米].

  3. Fubini定理
    ∑n=0∞∑米=0∞一个n米=∑米=0∞∑n=0∞一个n米
    至少满足以下条件之一即可:
    (i)一个n米≥0对所有人n.米
    (二)∑n=0∞∑米=0∞|一个n米|<∞
    ㈢∑米=0∞∑n=0∞|一个n米|<∞

数学代写|随机过程统计代写Stochastic process statistics代考|Homogeneous Random Walk

这里状态空间由下式给出

小号=…,−2,−1,0,1,2… p一世=p,q一世=q 对所有人 一世≥1.
这是一个不可约的 MC 因此,根据团结定理,考虑状态就足够了0只要。这n- 步复发概率是0是瞬态的∑n=0∞磷00(n)<∞和复发当先∑n=0∞磷00(n)=∞(由定理 2.5)

∑米=1∞(2米 米)p米q米≅∑米=1∞(4pq)米(圆周率米)1/2<∞ 如果 4pq<1=∞ 如果 4pq≥1(使用斯特林的近似为米 ! ≅2圆周率和−米米米+1/2 ) 4pq>1如果是不可能的4pq>(p+q)2然后0>(p−q)2.
因此
4pq<1如果p≠q

=1 如果 p=q=12. 
所以∑米=0∞p00(2米)收敛速度比几何级数快∑米(4pq)米如果p≠1/2.
因此随机游走是递归的p=12和瞬态当先p≠12.
我们在练习中展示了2.1一维的对称随机游走是循环的。类似地,可以证明在二维中对称随机游走是循环的。但是波利亚证明了ķ≥3尺寸对称随机游走是瞬态的。

数学代写|随机过程统计代写Stochastic process statistics代考 请认准statistics-lab™

统计代写请认准statistics-lab™. statistics-lab™为您的留学生涯保驾护航。

金融工程代写

金融工程是使用数学技术来解决金融问题。金融工程使用计算机科学、统计学、经济学和应用数学领域的工具和知识来解决当前的金融问题,以及设计新的和创新的金融产品。

非参数统计代写

非参数统计指的是一种统计方法,其中不假设数据来自于由少数参数决定的规定模型;这种模型的例子包括正态分布模型和线性回归模型。

广义线性模型代考

广义线性模型(GLM)归属统计学领域,是一种应用灵活的线性回归模型。该模型允许因变量的偏差分布有除了正态分布之外的其它分布。

术语 广义线性模型(GLM)通常是指给定连续和/或分类预测因素的连续响应变量的常规线性回归模型。它包括多元线性回归,以及方差分析和方差分析(仅含固定效应)。

有限元方法代写

有限元方法(FEM)是一种流行的方法,用于数值解决工程和数学建模中出现的微分方程。典型的问题领域包括结构分析、传热、流体流动、质量运输和电磁势等传统领域。

有限元是一种通用的数值方法,用于解决两个或三个空间变量的偏微分方程(即一些边界值问题)。为了解决一个问题,有限元将一个大系统细分为更小、更简单的部分,称为有限元。这是通过在空间维度上的特定空间离散化来实现的,它是通过构建对象的网格来实现的:用于求解的数值域,它有有限数量的点。边界值问题的有限元方法表述最终导致一个代数方程组。该方法在域上对未知函数进行逼近。[1] 然后将模拟这些有限元的简单方程组合成一个更大的方程系统,以模拟整个问题。然后,有限元通过变化微积分使相关的误差函数最小化来逼近一个解决方案。

tatistics-lab作为专业的留学生服务机构,多年来已为美国、英国、加拿大、澳洲等留学热门地的学生提供专业的学术服务,包括但不限于Essay代写,Assignment代写,Dissertation代写,Report代写,小组作业代写,Proposal代写,Paper代写,Presentation代写,计算机作业代写,论文修改和润色,网课代做,exam代考等等。写作范围涵盖高中,本科,研究生等海外留学全阶段,辐射金融,经济学,会计学,审计学,管理学等全球99%专业科目。写作团队既有专业英语母语作者,也有海外名校硕博留学生,每位写作老师都拥有过硬的语言能力,专业的学科背景和学术写作经验。我们承诺100%原创,100%专业,100%准时,100%满意。

随机分析代写


随机微积分是数学的一个分支,对随机过程进行操作。它允许为随机过程的积分定义一个关于随机过程的一致的积分理论。这个领域是由日本数学家伊藤清在第二次世界大战期间创建并开始的。

时间序列分析代写

随机过程,是依赖于参数的一组随机变量的全体,参数通常是时间。 随机变量是随机现象的数量表现,其时间序列是一组按照时间发生先后顺序进行排列的数据点序列。通常一组时间序列的时间间隔为一恒定值(如1秒,5分钟,12小时,7天,1年),因此时间序列可以作为离散时间数据进行分析处理。研究时间序列数据的意义在于现实中,往往需要研究某个事物其随时间发展变化的规律。这就需要通过研究该事物过去发展的历史记录,以得到其自身发展的规律。

回归分析代写

多元回归分析渐进(Multiple Regression Analysis Asymptotics)属于计量经济学领域,主要是一种数学上的统计分析方法,可以分析复杂情况下各影响因素的数学关系,在自然科学、社会和经济学等多个领域内应用广泛。

MATLAB代写

MATLAB 是一种用于技术计算的高性能语言。它将计算、可视化和编程集成在一个易于使用的环境中,其中问题和解决方案以熟悉的数学符号表示。典型用途包括:数学和计算算法开发建模、仿真和原型制作数据分析、探索和可视化科学和工程图形应用程序开发,包括图形用户界面构建MATLAB 是一个交互式系统,其基本数据元素是一个不需要维度的数组。这使您可以解决许多技术计算问题,尤其是那些具有矩阵和向量公式的问题,而只需用 C 或 Fortran 等标量非交互式语言编写程序所需的时间的一小部分。MATLAB 名称代表矩阵实验室。MATLAB 最初的编写目的是提供对由 LINPACK 和 EISPACK 项目开发的矩阵软件的轻松访问,这两个项目共同代表了矩阵计算软件的最新技术。MATLAB 经过多年的发展,得到了许多用户的投入。在大学环境中,它是数学、工程和科学入门和高级课程的标准教学工具。在工业领域,MATLAB 是高效研究、开发和分析的首选工具。MATLAB 具有一系列称为工具箱的特定于应用程序的解决方案。对于大多数 MATLAB 用户来说非常重要,工具箱允许您学习应用专业技术。工具箱是 MATLAB 函数(M 文件)的综合集合,可扩展 MATLAB 环境以解决特定类别的问题。可用工具箱的领域包括信号处理、控制系统、神经网络、模糊逻辑、小波、仿真等。

R语言代写问卷设计与分析代写
PYTHON代写回归分析与线性模型代写
MATLAB代写方差分析与试验设计代写
STATA代写机器学习/统计学习代写
SPSS代写计量经济学代写
EVIEWS代写时间序列分析代写
EXCEL代写深度学习代写
SQL代写各种数据建模与可视化代写

数学代写|随机过程统计代写Stochastic process statistics代考|STAT3021

如果你也在 怎样代写随机过程统计Stochastic process statistics这个学科遇到相关的难题,请随时右上角联系我们的24/7代写客服。

随机过程 用于表示在时间上发展的统计现象以及在处理这些现象时出现的理论模型,由于这些现象在许多领域都会遇到,因此这篇文章具有广泛的实际意义。

statistics-lab™ 为您的留学生涯保驾护航 在代写随机过程统计Stochastic process statistics方面已经树立了自己的口碑, 保证靠谱, 高质且原创的统计Statistics代写服务。我们的专家在代写随机过程统计Stochastic process statistics代写方面经验极为丰富,各种代写随机过程统计Stochastic process statistics相关的作业也就用不着说。

我们提供的随机过程统计Stochastic process statistics及其相关学科的代写,服务范围广, 其中包括但不限于:

  • Statistical Inference 统计推断
  • Statistical Computing 统计计算
  • Advanced Probability Theory 高等概率论
  • Advanced Mathematical Statistics 高等数理统计学
  • (Generalized) Linear Models 广义线性模型
  • Statistical Machine Learning 统计机器学习
  • Longitudinal Data Analysis 纵向数据分析
  • Foundations of Data Science 数据科学基础
数学代写|随机过程统计代写Stochastic process statistics代考|STAT3021

数学代写|随机过程统计代写Stochastic process statistics代考|Discrete Time Markov Chain

  1. Suppose $P$ is a stochastic matrix, then show that $P^{n}$ is also a stochastic matrix for all $n>1$.
  2. If $P^{n}$ is stochastic, is $P$ stochastic?
  3. Show that 1 is an eigenvalue if $A$ is a stochastic matrix, i.e.
    $$
    |\lambda I-A|=0 \Rightarrow \lambda=1 \text {. }
    $$
    Consider a sequence of trials with possible outcomes $E_{1}, E_{2}, \ldots, E_{k} \ldots$ To the pairs of outcomes $\left(E_{j}, E_{k}\right)$ we can associate some numbers (i.e. conditional probabilities) $P_{j k}$. The $\left{E_{k}\right}$ are referred to as the possible states of the system. Instead of saying that the $n$th trial results in $E_{k}$ one says that the $n$th step leads to $E_{k}$ or that $E_{k}$ is entered at the $n$th step.

We shall denote by $P_{j k}^{(n)}$ the probability of transition from $E_{j}$ to $E_{k}$ in exactly n steps i.e. the conditional probability of entering $E_{k}$ at the $n$th step from $E_{j}$. This is the sum of all the probabilities of all possible paths $E_{j} \rightarrow E_{j_{1}} \rightarrow \ldots E_{j_{n-1}} \rightarrow E_{k}$ of length $n$ starting at $E_{j}$. and ending at $E_{k}$
In particular,
$$
p_{j k}^{(1)}=p_{j k} .
$$
Theorem 2.2 (Chapman-Kolmogorov equation)
$$
p_{i j}^{(n)}=\sum_{k \varepsilon s} p_{i k}^{(n-1)} p_{k j}
$$
Proof
$$
\begin{aligned}
P_{i j}^{(n)} &=P\left[X_{n}=j \mid X_{0} \equiv i\right] \
&=\sum_{k E s} P\left[X_{n}=j\left|X_{n-1}=k, X_{0}=i\right| P\left[X_{n-1}=k \mid X_{0}=i\right]\right.\
&=\sum_{k \in s} P\left[X_{n}=j \mid X_{n-1}=k\right] P\left[X_{n-1}=k \mid X_{0}=i\right] \text { (by Markov property) } \
&=\sum_{k E s} P_{k j}^{(1)} P_{i k}^{(n-1)}=\sum_{k E s} P_{k j} P_{i k}^{(n-1)}
\end{aligned}
$$
Corollary For
$$
0 \leq m \leq n, p_{i j}^{(n)}=\sum_{k \in s} p_{i k}^{(n-m)} p_{k j}^{(m)}
$$

数学代写|随机过程统计代写Stochastic process statistics代考|A Few More Examples

(a) Independent trials
$P^{n}=P$ for all $n \geq 1$, where $p_{i j}=p_{j}$ i.e. all the rows are same.
(b) Success runs
Consider an infinite sequence of Bernoulli trials and at the $n$th trial the system is in the state $E_{j}$ if the last failure occurred at the trial number $n-j, j=0,1$, $2, \ldots$ and zero-th trial counts as failure. In other words, the index $j$ equals the length of uninterrupted run of successes ending at $n$th trial.
Here
$$
p_{i j}^{(n)}=\left{\begin{array}{l}
q p^{j} \text { for } j=0,1,2, \ldots, i+n-1 \
p^{j} \text { for } j=j+n \
0 \text { otherwise }
\end{array}\right.
$$
This follows either directly or from Chapman-Kolmogorov’s equation. It can be shown that $P^{n}$ converges to a matrix whose all elements in the column $j$ equals $q p^{j}$, where the transition matrix $P$ is given by
$$
P_{i j}=P\left(X_{n}=j \mid X_{n-1}=i\right)=\left{\begin{array}{l}
p \text { if } j=i+1 \
q \text { if } j=0 \
0 \text { otherwise }
\end{array}\right.
$$
(c) Two state M.C.
There are two possible states $E_{1}$ and $E_{2}$ in which the matrix of transition probability is of the form
$$
P=\left(\begin{array}{cc}
1-p & p \
a & 1-a
\end{array}\right), 0<p<1 \text { and } 0<a<1 .
$$
The system is said to be in state $E_{1}$ if a particle moves in the positive direction and in $E_{2}$ if the direction is negative.
(d) Random walk with absorbing barriers
Let the possible states be $E_{0}, E_{1}, E_{2}, \ldots, E_{k}$. Consider the matrix of transition probabilities

数学代写|随机过程统计代写Stochastic process statistics代考|Decomposition of state space

It may be possible that $p_{i j}=0, p_{i j}^{(2)}=0$ but $p_{i j}^{(3)}>0$. We say that the state $j$ is accessible from state $i$ if $p_{i j}^{(n)}>0$ for some $n>0$. In notation $i \rightarrow j$, i.e. $i$ leads to $j$. If $i \rightarrow j$ and $j \rightarrow i$, then $i$ and $j$ communicate and we denote this by $i \leftrightarrow j$.
Definition 2.4 The state $i$ is essential if $i \rightarrow j$ implies $i \leftarrow j$, i.e. if any state $j$ is accessible from $i$, then $i$ is accessible from that state. We shall let $\mathcal{S}$ denote the set of all essential states. States that are not essential are called inessential.
Lemma $2.1 \quad i \leftrightarrow j$ defines an equivalence relation on $\mathfrak{S}$, the class of essential states.
Proof $i \leftrightarrow i$ (reflexivity)
(i) Since for each $i, \sum_{j E s} p_{i j}=1$ there exists at least one $j$ for which $p_{i j}>0$. But if $i$ is essential then there exists $m \geq 1$ such that $p_{j i}^{(m)}>0$. So by ChapmenKolmogrov equation $p_{i i}^{(m+1)} \geq p_{i j} p_{j i}^{(m)}>0$.
(ii) $i \leftrightarrow j \Leftrightarrow j \leftrightarrow i$ (symmetry)
(iii) $i \leftrightarrow j$ and $j \leftrightarrow k \Rightarrow i \leftrightarrow k$ (transitivity)
Proof of (iii)
To prove $i \rightarrow k$, since $i \rightarrow j p_{j i}^{(n)}>0$ for some $n \geq 1$ and $j \rightarrow k, p_{j k}^{(m)}>0$ for some $m \geq 1$.
Claim: $p_{i k}^{(l)}>0$ for some $l \geq 1$
$$
0<p_{i j}^{(n)} p_{j k}^{(m)} \leq \sum_{j e s} p_{i j}^{(n)} p_{j k}^{(m)}=p_{i k}^{(n+m)} \text { (Chapman-Kolmogorov) }
$$
Taking $l=m+n$,
$$
i \rightarrow k \text { and similarly } k \rightarrow i \Rightarrow i \leftrightarrow k \text {. }
$$
By Lemma 2.1, i.e. $\mathfrak{J}=\cup{C(i)$, where $C(i)={j \in \mathcal{S} \mid i \leftrightarrow j}$ is called a communicating class, i.e. the class of essential states is partitioned into disjoint equivalent classes (communicating classes).
Definition $2.5$
A Markov chain is called irreducible (or ergodic) if there is only one communicating class, i.e. if all states communicate with each other or every state can be reached from every other state.
Definition $2.6$
A subset $C$ of $S$ is called closed (or transient) if it is impossible to leave $C$ in one step i.e. $p_{i j}=0$ for all $i \in C$ and all $j \notin C$.

数学代写|随机过程统计代写Stochastic process statistics代考|STAT3021

随机过程统计代考

数学代写|随机过程统计代写Stochastic process statistics代考|Discrete Time Markov Chain

  1. 认为磷是一个随机矩阵,那么证明磷n也是一个随机矩阵n>1.
  2. 如果磷n是随机的,是磷随机?
  3. 证明 1 是一个特征值,如果一个是一个随机矩阵,即
    |λ我−一个|=0⇒λ=1. 
    考虑一系列具有可能结果的试验和1,和2,…,和ķ…到成对的结果(和j,和ķ)我们可以关联一些数字(即条件概率)磷jķ. 这\left{E_{k}\right}\left{E_{k}\right}称为系统的可能状态。而不是说n试验结果和ķ有人说n这一步导致和ķ或者那个和ķ被输入在n第一步。

我们将表示为磷jķ(n)从转换的概率和j至和ķ在恰好 n 个步骤中,即进入的条件概率和ķ在n从第和j. 这是所有可能路径的所有概率的总和和j→和j1→…和jn−1→和ķ长度n开始于和j. 并结束于和ķ
尤其是,

pjķ(1)=pjķ.
定理 2.2(Chapman-Kolmogorov 方程)

p一世j(n)=∑ķesp一世ķ(n−1)pķj
证明

磷一世j(n)=磷[Xn=j∣X0≡一世] =∑ķ和s磷[Xn=j|Xn−1=ķ,X0=一世|磷[Xn−1=ķ∣X0=一世] =∑ķ∈s磷[Xn=j∣Xn−1=ķ]磷[Xn−1=ķ∣X0=一世] (由马尔可夫财产)  =∑ķ和s磷ķj(1)磷一世ķ(n−1)=∑ķ和s磷ķj磷一世ķ(n−1)
推论

0≤米≤n,p一世j(n)=∑ķ∈sp一世ķ(n−米)pķj(米)

数学代写|随机过程统计代写Stochastic process statistics代考|A Few More Examples

(a) 独立审判
磷n=磷对所有人n≥1, 在哪里p一世j=pj即所有行都是相同的。
(b) 成功运行
考虑一个无限序列的伯努利试验,并且在n系统处于试用状态和j如果最后一次失败发生在试用号n−j,j=0,1, 2,…并且第零次尝试算作失败。换句话说,索引j等于连续成功运行的长度,结束于n审判。
这里
$$
p_{ij}^{(n)}=\left{

qpj 为了 j=0,1,2,…,一世+n−1 pj 为了 j=j+n 0 否则 \正确的。

吨H一世sF○ll○在s和一世吨H和rd一世r和C吨l是○rFr○米CH一个p米一个n−ķ○l米○G○r○在′s和q在一个吨一世○n.我吨C一个nb和sH○在n吨H一个吨$磷n$C○n在和rG和s吨○一个米一个吨r一世X在H○s和一个ll和l和米和n吨s一世n吨H和C○l在米n$j$和q在一个ls$qpj$,在H和r和吨H和吨r一个ns一世吨一世○n米一个吨r一世X$磷$一世sG一世在和nb是
P_{ij}=P\left(X_{n}=j \mid X_{n-1}=i\right)=\left{

p 如果 j=一世+1 q 如果 j=0 0 否则 \正确的。

(C)吨在○s吨一个吨和米.C.吨H和r和一个r和吨在○p○ss一世bl和s吨一个吨和s$和1$一个nd$和2$一世n在H一世CH吨H和米一个吨r一世X○F吨r一个ns一世吨一世○npr○b一个b一世l一世吨是一世s○F吨H和F○r米
P=\左(

1−pp 一个1−一个\right), 0<p<1 \text { 和 } 0<a<1 。
$$
系统据说处于状态和1如果一个粒子在正方向和和2如果方向为负。
(d) 具有吸收障碍的随机游走
让可能的状态为和0,和1,和2,…,和ķ. 考虑转移概率矩阵

数学代写|随机过程统计代写Stochastic process statistics代考|Decomposition of state space

有可能是p一世j=0,p一世j(2)=0但p一世j(3)>0. 我们说国家j可从状态访问一世如果p一世j(n)>0对于一些n>0. 在符号一世→j, IE一世导致j. 如果一世→j和j→一世, 然后一世和j沟通,我们用一世↔j.
定义 2.4 状态一世是必不可少的,如果一世→j暗示一世←j,即如果有任何状态j可从一世, 然后一世可以从该状态访问。我们要让小号表示所有基本状态的集合。非必要状态称为非必要状态。
引理2.1一世↔j定义一个等价关系小号,基本状态类。
证明一世↔一世(自反性)
(i) 因为对于每个一世,∑j和sp一世j=1至少存在一个j为此p一世j>0. 但如果一世是必不可少的,那么存在米≥1这样pj一世(米)>0. 所以由 ChapmenKolmogrov 方程p一世一世(米+1)≥p一世jpj一世(米)>0.
(二)一世↔j⇔j↔一世(对称)
(iii)一世↔j和j↔ķ⇒一世↔ķ(及物性)
证明(iii)
证明一世→ķ, 自从一世→jpj一世(n)>0对于一些n≥1和j→ķ,pjķ(米)>0对于一些米≥1.
宣称:p一世ķ(l)>0对于一些l≥1

0<p一世j(n)pjķ(米)≤∑j和sp一世j(n)pjķ(米)=p一世ķ(n+米) (查普曼-科尔莫哥洛夫) 
服用l=米+n,

一世→ķ 同样地 ķ→一世⇒一世↔ķ. 
根据引理 2.1,即 $\mathfrak{J}=\cup{C(i),在H和r和C(i)={j \in \mathcal{S} \mid i \leftrightarrow j}一世sC一个ll和d一个C○米米在n一世C一个吨一世nGCl一个ss,一世.和.吨H和Cl一个ss○F和ss和n吨一世一个ls吨一个吨和s一世sp一个r吨一世吨一世○n和d一世n吨○d一世sj○一世n吨和q在一世在一个l和n吨Cl一个ss和s(C○米米在n一世C一个吨一世nGCl一个ss和s).D和F一世n一世吨一世○n2.5一个米一个rķ○在CH一个一世n一世sC一个ll和d一世rr和d在C一世bl和(○r和rG○d一世C)一世F吨H和r和一世s○nl是○n和C○米米在n一世C一个吨一世nGCl一个ss,一世.和.一世F一个lls吨一个吨和sC○米米在n一世C一个吨和在一世吨H和一个CH○吨H和r○r和在和r是s吨一个吨和C一个nb和r和一个CH和dFr○米和在和r是○吨H和rs吨一个吨和.D和F一世n一世吨一世○n2.6一个s在bs和吨C○F小号一世sC一个ll和dCl○s和d(○r吨r一个ns一世和n吨)一世F一世吨一世s一世米p○ss一世bl和吨○l和一个在和C一世n○n和s吨和p一世.和.p_{ij}=0F○r一个ll我\在C一个nd一个llj \notin C$。

数学代写|随机过程统计代写Stochastic process statistics代考 请认准statistics-lab™

统计代写请认准statistics-lab™. statistics-lab™为您的留学生涯保驾护航。

金融工程代写

金融工程是使用数学技术来解决金融问题。金融工程使用计算机科学、统计学、经济学和应用数学领域的工具和知识来解决当前的金融问题,以及设计新的和创新的金融产品。

非参数统计代写

非参数统计指的是一种统计方法,其中不假设数据来自于由少数参数决定的规定模型;这种模型的例子包括正态分布模型和线性回归模型。

广义线性模型代考

广义线性模型(GLM)归属统计学领域,是一种应用灵活的线性回归模型。该模型允许因变量的偏差分布有除了正态分布之外的其它分布。

术语 广义线性模型(GLM)通常是指给定连续和/或分类预测因素的连续响应变量的常规线性回归模型。它包括多元线性回归,以及方差分析和方差分析(仅含固定效应)。

有限元方法代写

有限元方法(FEM)是一种流行的方法,用于数值解决工程和数学建模中出现的微分方程。典型的问题领域包括结构分析、传热、流体流动、质量运输和电磁势等传统领域。

有限元是一种通用的数值方法,用于解决两个或三个空间变量的偏微分方程(即一些边界值问题)。为了解决一个问题,有限元将一个大系统细分为更小、更简单的部分,称为有限元。这是通过在空间维度上的特定空间离散化来实现的,它是通过构建对象的网格来实现的:用于求解的数值域,它有有限数量的点。边界值问题的有限元方法表述最终导致一个代数方程组。该方法在域上对未知函数进行逼近。[1] 然后将模拟这些有限元的简单方程组合成一个更大的方程系统,以模拟整个问题。然后,有限元通过变化微积分使相关的误差函数最小化来逼近一个解决方案。

tatistics-lab作为专业的留学生服务机构,多年来已为美国、英国、加拿大、澳洲等留学热门地的学生提供专业的学术服务,包括但不限于Essay代写,Assignment代写,Dissertation代写,Report代写,小组作业代写,Proposal代写,Paper代写,Presentation代写,计算机作业代写,论文修改和润色,网课代做,exam代考等等。写作范围涵盖高中,本科,研究生等海外留学全阶段,辐射金融,经济学,会计学,审计学,管理学等全球99%专业科目。写作团队既有专业英语母语作者,也有海外名校硕博留学生,每位写作老师都拥有过硬的语言能力,专业的学科背景和学术写作经验。我们承诺100%原创,100%专业,100%准时,100%满意。

随机分析代写


随机微积分是数学的一个分支,对随机过程进行操作。它允许为随机过程的积分定义一个关于随机过程的一致的积分理论。这个领域是由日本数学家伊藤清在第二次世界大战期间创建并开始的。

时间序列分析代写

随机过程,是依赖于参数的一组随机变量的全体,参数通常是时间。 随机变量是随机现象的数量表现,其时间序列是一组按照时间发生先后顺序进行排列的数据点序列。通常一组时间序列的时间间隔为一恒定值(如1秒,5分钟,12小时,7天,1年),因此时间序列可以作为离散时间数据进行分析处理。研究时间序列数据的意义在于现实中,往往需要研究某个事物其随时间发展变化的规律。这就需要通过研究该事物过去发展的历史记录,以得到其自身发展的规律。

回归分析代写

多元回归分析渐进(Multiple Regression Analysis Asymptotics)属于计量经济学领域,主要是一种数学上的统计分析方法,可以分析复杂情况下各影响因素的数学关系,在自然科学、社会和经济学等多个领域内应用广泛。

MATLAB代写

MATLAB 是一种用于技术计算的高性能语言。它将计算、可视化和编程集成在一个易于使用的环境中,其中问题和解决方案以熟悉的数学符号表示。典型用途包括:数学和计算算法开发建模、仿真和原型制作数据分析、探索和可视化科学和工程图形应用程序开发,包括图形用户界面构建MATLAB 是一个交互式系统,其基本数据元素是一个不需要维度的数组。这使您可以解决许多技术计算问题,尤其是那些具有矩阵和向量公式的问题,而只需用 C 或 Fortran 等标量非交互式语言编写程序所需的时间的一小部分。MATLAB 名称代表矩阵实验室。MATLAB 最初的编写目的是提供对由 LINPACK 和 EISPACK 项目开发的矩阵软件的轻松访问,这两个项目共同代表了矩阵计算软件的最新技术。MATLAB 经过多年的发展,得到了许多用户的投入。在大学环境中,它是数学、工程和科学入门和高级课程的标准教学工具。在工业领域,MATLAB 是高效研究、开发和分析的首选工具。MATLAB 具有一系列称为工具箱的特定于应用程序的解决方案。对于大多数 MATLAB 用户来说非常重要,工具箱允许您学习应用专业技术。工具箱是 MATLAB 函数(M 文件)的综合集合,可扩展 MATLAB 环境以解决特定类别的问题。可用工具箱的领域包括信号处理、控制系统、神经网络、模糊逻辑、小波、仿真等。

R语言代写问卷设计与分析代写
PYTHON代写回归分析与线性模型代写
MATLAB代写方差分析与试验设计代写
STATA代写机器学习/统计学习代写
SPSS代写计量经济学代写
EVIEWS代写时间序列分析代写
EXCEL代写深度学习代写
SQL代写各种数据建模与可视化代写

数学代写|随机过程统计代写Stochastic process statistics代考|STAT3921

如果你也在 怎样代写随机过程统计Stochastic process statistics这个学科遇到相关的难题,请随时右上角联系我们的24/7代写客服。

随机过程 用于表示在时间上发展的统计现象以及在处理这些现象时出现的理论模型,由于这些现象在许多领域都会遇到,因此这篇文章具有广泛的实际意义。

statistics-lab™ 为您的留学生涯保驾护航 在代写随机过程统计Stochastic process statistics方面已经树立了自己的口碑, 保证靠谱, 高质且原创的统计Statistics代写服务。我们的专家在代写随机过程统计Stochastic process statistics代写方面经验极为丰富,各种代写随机过程统计Stochastic process statistics相关的作业也就用不着说。

我们提供的随机过程统计Stochastic process statistics及其相关学科的代写,服务范围广, 其中包括但不限于:

  • Statistical Inference 统计推断
  • Statistical Computing 统计计算
  • Advanced Probability Theory 高等概率论
  • Advanced Mathematical Statistics 高等数理统计学
  • (Generalized) Linear Models 广义线性模型
  • Statistical Machine Learning 统计机器学习
  • Longitudinal Data Analysis 纵向数据分析
  • Foundations of Data Science 数据科学基础
数学代写|随机过程统计代写Stochastic process statistics代考|STAT3921

数学代写|随机过程统计代写Stochastic process statistics代考|Ergodicity

The behavior in which sample averages formed from a process converge to some underlying parameter of the process is termed ergodic. To make inference about the underlying laws governing an ergodic process, one need not observe separate independent replications of entire processes or sample paths. Instead, one need only observe a single realization of the process, but over a sufficiently long span of time. Thus, it is an important practical problem to determine conditions that lead to a stationary process being ergodic. The theory of stationary processes has a prime goal the clarification of ergodic behavior and the prediction problem for processes falling in the wide range of extremeties.

In covariance stationary process usually the added condition that $E\left(X_{t}\right)$ does not depend on $t$ is imposed. But it should be noted that in order for a stochastic process with $E\left(X_{1}^{2}\right)<\infty$ to be covariance stationary it is not necessary that its mean function $m(t)=E\left(X_{t}\right)$ be a constant. Consider the example: $X(t)=$ $\cos \left(\frac{2 \pi t}{L}\right)+Y(t)$, where $Y(t)=N(t+L)-N(t),{N(t), t \geq 0}$ be a Poisson process with intensity parameter $\lambda$ (to be defined in Chapter 7 ) and $L$ is a positive constant. Its mean function $m(t)=E\left(X_{t}\right)=\lambda(t+L)-\lambda(t)+\cos \left(\frac{2 \pi t}{L}\right)$ is functionally dependent on $t$. But $$ \begin{aligned} \operatorname{Cov}(X(t), X(s)) &=\operatorname{Cov}(Y(t), Y(s)) \ &=\left{\begin{array}{rr} \lambda(L-|t-s|) & \text { if }|t-s| \leq L \ 0 & \text { if }|t-s|>L
\end{array}\right.
\end{aligned}
$$
depends on $t-s$ only.

数学代写|随机过程统计代写Stochastic process statistics代考|Exercises and Complements

Exercise 1.1 Let $a, b, c$ be independent r.v.’s uniformly distributed on $[0,1]$. What is the probability that $a x^{2}+b x+c$ has real roots?

Exercise $1.2$ Let $X$ be a Poisson r.v. with parameter $\lambda>0$. Suppose $\lambda$ itself is a r.v. following a gamma distribution with density $f(\lambda)=\frac{1}{\sqrt{n}} \lambda^{n-1} e^{-\lambda} \cdot \lambda \geq 0$. Show that $P(X=k)=\frac{\sqrt{k+n}}{\sqrt{n} \sqrt{k+1}}(1 / 2)^{k+n}, k \geq 0$ (note that when $n$ is a positive integer $X$ is negative binomial with $p=1 / 2)$.

Exercise 1.3 The following experiment is performed. An observation is made of a Poisson r.v. $X$ with parameter $\lambda$. Then a binomial event $Y$ with probability $p$ of success is repeated $X$ number of times and $Y$ successes are observed. What is the distribution of $Y$ ?

Exercise 1.4 Let $\left{X_{t}, t \geq 0\right}$ be a continuous time stochastic process with independent increments. Also $P\left(X_{0}=0\right)=1$. If $\phi(\theta, t-u)$ is the characteristic function of a single increment i.e.
$$
\phi(\theta, t-u)=E\left[\exp \left(i \theta\left(X_{t}-X_{u}\right)\right)\right],
$$
prove that the joint characteristic function of $X_{t_{1}}, X_{t_{2}}, \ldots, X_{t_{n}}$ where $t_{1}<t_{2}<\ldots<t_{n}$ is
$$
\phi\left(\sum_{j=1}^{n} \theta_{j}, t_{1}\right) \phi\left(\sum_{j=2}^{n} \theta_{j}, t_{2}-t_{1}\right) \ldots \phi\left(\theta_{n}, t_{n}-t_{n-1}\right) .
$$

Exercise 1.5 Prove that every continuous parameter Stochastic process with independent increments is a Markov process.

Exercise 1.6 Let $T$ be a nonnegative discrete random variable. Prove that $T$ has a geometric distribution iff
$$
P[T>x+y \mid T>x]=P[T>y] \text { for all integers } x, y \geq 0 \text {. }
$$
Exercise $1.7$ Let $T$ be a non-negative continuous random variable. Prove that $T$ has an exponential distribution iff $P[7>x+y \mid T>x]=P[T>y]$.

Exercise $1.8$ Let $T$ he a nonnegative random variable such that $X(T)$ is a stochastic process and for a fixed valuc of $T$, say $t, X(t)$ has a gamma density
$$
f_{x(t)}(u)=\frac{1}{\sqrt{\alpha t}}(\lambda u)^{\alpha t-1} e^{-\lambda u}, \alpha>0, \lambda>0 .
$$
Assume that the distribution of $T$ is $F(t)=P[T \leq t]$.
(a) Derive an expression for $E\left[e^{-r X(T)}\right]$
(b) Prove that $T$ has a negative binomial distribution, i.e.
$$
P\left[T=\frac{\beta+k}{\alpha}\right]=\left(\begin{array}{c}
\beta+k-1 \
k
\end{array}\right) p^{\beta}(1-p)^{k}, \beta>0
$$
$$
1>p=1-q>0 . k=0.1 .2, \ldots
$$
then $X(T)$ also has a gamma density. Derive its parameters.
(c) Prove that if $X(T)$ has a gamma distribution, i.e.
$E\left|e^{-w(T)}\right|=\left(\frac{\mu}{\mu+s}\right)^{\beta}$ where $\beta>0, \lambda>\mu \geq 0$, then conversely, $T$ has a negative binomial distribution. Determine its parameters.

数学代写|随机过程统计代写Stochastic process statistics代考|Definition and Transition Probabilities

Here $S=$ a countable set, $T={0,1,2, \ldots},\left{X_{n}, n \geq 0\right}$ is a stochastic process satisfying $P\left[X_{n+1}=j \mid X_{0}=i_{0}, X_{1}=i_{1}, \ldots, X_{n}=i_{n}\right]=P\left[X_{n+1}=j \mid X_{n}=i_{n}\right]$, the Markov property. Then the stochastic process $\left{X_{n}, n \geq 0\right}$ is called a Markov chain (M.C.). We shall assume that the M.C. is stationary i.e. $P\left[X_{n+1}=j \mid X_{n}=\right.$ $i]=p_{i j}$ is independent of $n$ for all $i, j \in, S$. Let $P=\left(P_{i j}\right) ; i, j \in S$ be a finite or countably infinite dimensional matrix with elements $p_{i j}$.

The matrix $P$ is called the one step transition matrix of the M.C. or simply the Transition matrix or the Probability matrix of the M.C.

Example (Random Walk) A random walk on the (real) line is a Markov chain such that
$$
p_{j k}=0 \text { if } k \neq j-1 \text { or } j+1 .
$$
Transition is possible only to neighbouring states (from $j$ to $j-1$ and $j+1$ ). Here state space is
$$
S={\ldots,-3,-2,-1,0,1,2,3, \ldots} .
$$
Theorem 2.1 The Markov chain $\left{X_{n}, n \geq 0\right}$ is completely determined by the transition matrix $P$ and the initial distribution $\left{p_{k}\right}$, defined as $P\left[X_{0}=k\right]=p_{k} \geq 0$, $\sum_{k \in s} p_{k}=1$
Proof
$$
\begin{aligned}
P\left[X_{0}\right.&\left.=i_{0}, X_{1}=i_{i}, \ldots, X_{n}=i_{n}\right] \
&=P\left[X_{n}=i_{n} \mid X_{n-1}=i_{n-1}, X_{n-2}=i_{n-2}, \ldots, X_{1}=i_{1} \ldots X_{0}=i_{0}\right] \
P\left[X_{n-1}\right.&\left.=i_{n-1}, X_{n-2}=i_{n-2}, \ldots, X_{1}=i_{1}, X_{0}=i_{0}\right] \
&=P\left[X_{n}=i_{n} \mid X_{n-1}=i_{n-1}\right] P\left[X_{n-1}=i_{n-1}, \ldots, X_{0}=i_{0}\right] \
&=p_{i_{n-1} i_{n}} p_{i_{n-2} i_{n-1}} P\left[X_{n-2}=i_{n-2}, \ldots, X_{0}=i_{0}\right] \
&=p_{i_{n-1} i_{n}} p_{i_{n-2} i_{n-1}} \ldots p_{i_{i} i_{2}} p_{i_{0} i_{1}} p_{i_{0}} \text { (by induction). }
\end{aligned}
$$
Definition 2.1 A vector $u=\left(u_{1}, u_{2}, \ldots, u_{n}\right)$ is called a probability vector if the components are non-negative and their sum is one.

数学代写|随机过程统计代写Stochastic process statistics代考|STAT3921

随机过程统计代考

数学代写|随机过程统计代写Stochastic process statistics代考|Ergodicity

由一个过程形成的样本平均值收敛到该过程的某个基本参数的行为称为遍历。要推断支配遍历过程的基本规律,不需要观察整个过程或样本路径的单独独立复制。取而代之的是,人们只需要观察该过程的一次实现,但要经过足够长的时间跨度。因此,确定导致平稳过程遍历的条件是一个重要的实际问题。平稳过程理论的主要目标是阐明遍历行为和对处于广泛极端范围内的过程的预测问题。

在协方差平稳过程中,通常添加的条件是和(X吨)不依赖于吨被强加。但应该注意的是,为了使随机过程具有和(X12)<∞是协方差平稳的,它的平均函数没有必要米(吨)=和(X吨)成为一个常数。考虑这个例子:X(吨)= 因⁡(2圆周率吨大号)+是(吨), 在哪里是(吨)=ñ(吨+大号)−ñ(吨),ñ(吨),吨≥0是具有强度参数的泊松过程λ(将在第 7 章中定义)和大号是一个正常数。它的平均函数米(吨)=和(X吨)=λ(吨+大号)−λ(吨)+因⁡(2圆周率吨大号)在功能上取决于吨. 但是 $$ \begin{aligned} \operatorname{Cov}(X(t), X(s)) &=\operatorname{Cov}(Y(t), Y(s)) \ &=\left{

λ(大号−|吨−s|) 如果 |吨−s|≤大号 0 如果 |吨−s|>大号\正确的。
\end{aligned}
$$
取决于吨−s只要。

数学代写|随机过程统计代写Stochastic process statistics代考|Exercises and Complements

练习 1.1 让一个,b,C是独立的 rv 均匀分布在[0,1]. 发生的概率是多少一个X2+bX+C有真正的根源吗?

锻炼1.2让X是带参数的泊松 rvλ>0. 认为λ本身是一个 rv 遵循具有密度的伽马分布F(λ)=1nλn−1和−λ⋅λ≥0. 显示磷(X=ķ)=ķ+nnķ+1(1/2)ķ+n,ķ≥0(请注意,当n是一个正整数X是负二项式p=1/2).

练习 1.3 进行以下实验。观察泊松 rvX带参数λ. 然后是二项式事件是有概率p成功的重复X次数和是观察到成功。什么是分布是 ?

练习 1.4 让\left{X_{t}, t \geq 0\right}\left{X_{t}, t \geq 0\right}是一个具有独立增量的连续时间随机过程。还磷(X0=0)=1. 如果φ(θ,吨−在)是单个增量的特征函数,即

φ(θ,吨−在)=和[经验⁡(一世θ(X吨−X在))],
证明联合特征函数X吨1,X吨2,…,X吨n在哪里吨1<吨2<…<吨n是

φ(∑j=1nθj,吨1)φ(∑j=2nθj,吨2−吨1)…φ(θn,吨n−吨n−1).

练习 1.5 证明每个具有独立增量的连续参数随机过程都是马尔可夫过程。

练习 1.6 让吨是一个非负离散随机变量。证明吨有一个几何分布iff

磷[吨>X+是∣吨>X]=磷[吨>是] 对于所有整数 X,是≥0. 
锻炼1.7让吨为非负连续随机变量。证明吨具有指数分布 iff磷[7>X+是∣吨>X]=磷[吨>是].

锻炼1.8让吨他是一个非负随机变量,使得X(吨)是一个随机过程,对于固定值吨, 说吨,X(吨)有一个伽马密度

FX(吨)(在)=1一个吨(λ在)一个吨−1和−λ在,一个>0,λ>0.
假设分布吨是F(吨)=磷[吨≤吨].
(a) 导出表达式和[和−rX(吨)]
(b) 证明吨具有负二项分布,即

磷[吨=b+ķ一个]=(b+ķ−1 ķ)pb(1−p)ķ,b>0

1>p=1−q>0.ķ=0.1.2,…
然后X(吨)也有伽马密度。导出其参数。
(c) 证明如果X(吨)具有伽马分布,即
和|和−在(吨)|=(μμ+s)b在哪里b>0,λ>μ≥0,然后反过来,吨具有负二项分布。确定其参数。

数学代写|随机过程统计代写Stochastic process statistics代考|Definition and Transition Probabilities

这里小号=可数集,T={0,1,2, \ldots},\left{X_{n}, n \geq 0\right}T={0,1,2, \ldots},\left{X_{n}, n \geq 0\right}是一个满足的随机过程磷[Xn+1=j∣X0=一世0,X1=一世1,…,Xn=一世n]=磷[Xn+1=j∣Xn=一世n],马尔可夫性质。然后是随机过程\left{X_{n}, n \geq 0\right}\left{X_{n}, n \geq 0\right}称为马尔可夫链(MC)。我们将假设 MC 是静止的,即磷[Xn+1=j∣Xn= 一世]=p一世j独立于n对所有人一世,j∈,小号. 让磷=(磷一世j);一世,j∈小号是具有元素的有限或可数无限维矩阵p一世j.

矩阵磷称为 MC 的一步转移矩阵或简称为 MC 的转移矩阵或概率矩阵

示例(随机游走)(真实)线上的随机游走是马尔可夫链,使得

pjķ=0 如果 ķ≠j−1 或者 j+1.
只能向邻国过渡(从j至j−1和j+1)。这里的状态空间是

小号=…,−3,−2,−1,0,1,2,3,….
定理 2.1 马尔可夫链\left{X_{n}, n \geq 0\right}\left{X_{n}, n \geq 0\right}完全由转移矩阵决定磷和初始分布\左{p_{k}\右}\左{p_{k}\右}, 定义为磷[X0=ķ]=pķ≥0, ∑ķ∈spķ=1
证明

磷[X0=一世0,X1=一世一世,…,Xn=一世n] =磷[Xn=一世n∣Xn−1=一世n−1,Xn−2=一世n−2,…,X1=一世1…X0=一世0] 磷[Xn−1=一世n−1,Xn−2=一世n−2,…,X1=一世1,X0=一世0] =磷[Xn=一世n∣Xn−1=一世n−1]磷[Xn−1=一世n−1,…,X0=一世0] =p一世n−1一世np一世n−2一世n−1磷[Xn−2=一世n−2,…,X0=一世0] =p一世n−1一世np一世n−2一世n−1…p一世一世一世2p一世0一世1p一世0 (通过感应)。 
定义 2.1 向量在=(在1,在2,…,在n)如果分量是非负的并且它们的和为 1,则称为概率向量。

数学代写|随机过程统计代写Stochastic process statistics代考 请认准statistics-lab™

统计代写请认准statistics-lab™. statistics-lab™为您的留学生涯保驾护航。

金融工程代写

金融工程是使用数学技术来解决金融问题。金融工程使用计算机科学、统计学、经济学和应用数学领域的工具和知识来解决当前的金融问题,以及设计新的和创新的金融产品。

非参数统计代写

非参数统计指的是一种统计方法,其中不假设数据来自于由少数参数决定的规定模型;这种模型的例子包括正态分布模型和线性回归模型。

广义线性模型代考

广义线性模型(GLM)归属统计学领域,是一种应用灵活的线性回归模型。该模型允许因变量的偏差分布有除了正态分布之外的其它分布。

术语 广义线性模型(GLM)通常是指给定连续和/或分类预测因素的连续响应变量的常规线性回归模型。它包括多元线性回归,以及方差分析和方差分析(仅含固定效应)。

有限元方法代写

有限元方法(FEM)是一种流行的方法,用于数值解决工程和数学建模中出现的微分方程。典型的问题领域包括结构分析、传热、流体流动、质量运输和电磁势等传统领域。

有限元是一种通用的数值方法,用于解决两个或三个空间变量的偏微分方程(即一些边界值问题)。为了解决一个问题,有限元将一个大系统细分为更小、更简单的部分,称为有限元。这是通过在空间维度上的特定空间离散化来实现的,它是通过构建对象的网格来实现的:用于求解的数值域,它有有限数量的点。边界值问题的有限元方法表述最终导致一个代数方程组。该方法在域上对未知函数进行逼近。[1] 然后将模拟这些有限元的简单方程组合成一个更大的方程系统,以模拟整个问题。然后,有限元通过变化微积分使相关的误差函数最小化来逼近一个解决方案。

tatistics-lab作为专业的留学生服务机构,多年来已为美国、英国、加拿大、澳洲等留学热门地的学生提供专业的学术服务,包括但不限于Essay代写,Assignment代写,Dissertation代写,Report代写,小组作业代写,Proposal代写,Paper代写,Presentation代写,计算机作业代写,论文修改和润色,网课代做,exam代考等等。写作范围涵盖高中,本科,研究生等海外留学全阶段,辐射金融,经济学,会计学,审计学,管理学等全球99%专业科目。写作团队既有专业英语母语作者,也有海外名校硕博留学生,每位写作老师都拥有过硬的语言能力,专业的学科背景和学术写作经验。我们承诺100%原创,100%专业,100%准时,100%满意。

随机分析代写


随机微积分是数学的一个分支,对随机过程进行操作。它允许为随机过程的积分定义一个关于随机过程的一致的积分理论。这个领域是由日本数学家伊藤清在第二次世界大战期间创建并开始的。

时间序列分析代写

随机过程,是依赖于参数的一组随机变量的全体,参数通常是时间。 随机变量是随机现象的数量表现,其时间序列是一组按照时间发生先后顺序进行排列的数据点序列。通常一组时间序列的时间间隔为一恒定值(如1秒,5分钟,12小时,7天,1年),因此时间序列可以作为离散时间数据进行分析处理。研究时间序列数据的意义在于现实中,往往需要研究某个事物其随时间发展变化的规律。这就需要通过研究该事物过去发展的历史记录,以得到其自身发展的规律。

回归分析代写

多元回归分析渐进(Multiple Regression Analysis Asymptotics)属于计量经济学领域,主要是一种数学上的统计分析方法,可以分析复杂情况下各影响因素的数学关系,在自然科学、社会和经济学等多个领域内应用广泛。

MATLAB代写

MATLAB 是一种用于技术计算的高性能语言。它将计算、可视化和编程集成在一个易于使用的环境中,其中问题和解决方案以熟悉的数学符号表示。典型用途包括:数学和计算算法开发建模、仿真和原型制作数据分析、探索和可视化科学和工程图形应用程序开发,包括图形用户界面构建MATLAB 是一个交互式系统,其基本数据元素是一个不需要维度的数组。这使您可以解决许多技术计算问题,尤其是那些具有矩阵和向量公式的问题,而只需用 C 或 Fortran 等标量非交互式语言编写程序所需的时间的一小部分。MATLAB 名称代表矩阵实验室。MATLAB 最初的编写目的是提供对由 LINPACK 和 EISPACK 项目开发的矩阵软件的轻松访问,这两个项目共同代表了矩阵计算软件的最新技术。MATLAB 经过多年的发展,得到了许多用户的投入。在大学环境中,它是数学、工程和科学入门和高级课程的标准教学工具。在工业领域,MATLAB 是高效研究、开发和分析的首选工具。MATLAB 具有一系列称为工具箱的特定于应用程序的解决方案。对于大多数 MATLAB 用户来说非常重要,工具箱允许您学习应用专业技术。工具箱是 MATLAB 函数(M 文件)的综合集合,可扩展 MATLAB 环境以解决特定类别的问题。可用工具箱的领域包括信号处理、控制系统、神经网络、模糊逻辑、小波、仿真等。

R语言代写问卷设计与分析代写
PYTHON代写回归分析与线性模型代写
MATLAB代写方差分析与试验设计代写
STATA代写机器学习/统计学习代写
SPSS代写计量经济学代写
EVIEWS代写时间序列分析代写
EXCEL代写深度学习代写
SQL代写各种数据建模与可视化代写

数学代写|随机过程统计代写Stochastic process statistics代考|STAT4061

如果你也在 怎样代写随机过程统计Stochastic process statistics这个学科遇到相关的难题,请随时右上角联系我们的24/7代写客服。

随机过程 用于表示在时间上发展的统计现象以及在处理这些现象时出现的理论模型,由于这些现象在许多领域都会遇到,因此这篇文章具有广泛的实际意义。

statistics-lab™ 为您的留学生涯保驾护航 在代写随机过程统计Stochastic process statistics方面已经树立了自己的口碑, 保证靠谱, 高质且原创的统计Statistics代写服务。我们的专家在代写随机过程统计Stochastic process statistics代写方面经验极为丰富,各种代写随机过程统计Stochastic process statistics相关的作业也就用不着说。

我们提供的随机过程统计Stochastic process statistics及其相关学科的代写,服务范围广, 其中包括但不限于:

  • Statistical Inference 统计推断
  • Statistical Computing 统计计算
  • Advanced Probability Theory 高等概率论
  • Advanced Mathematical Statistics 高等数理统计学
  • (Generalized) Linear Models 广义线性模型
  • Statistical Machine Learning 统计机器学习
  • Longitudinal Data Analysis 纵向数据分析
  • Foundations of Data Science 数据科学基础
数学代写|随机过程统计代写Stochastic process statistics代考|STAT4061

数学代写|随机过程统计代写Stochastic process statistics代考|Notion of Stochastic Processes

Loosely speaking, the mathematical description of a random phenomenon as it changes in time is a stochastic process. Since the last century there has been greater realisation that stochastic (or non-deterministic) models are more realistic than deterministic models in many situations. Observations taken at different time points rather than those taken at a fixed period of time began to draw the attention of scientists. The physicists and communication engineers played a leading role in the development of dynamic indeterminism. Many a phenomenon occurring in physical and life sciences are studied not only as a random phenomenon but also as one changing with time or space. Similar considerations are also made in other areas such as social sciences, economics and management sciences, and so on. The scope of applications of stochastic processes which are functions of time or space or both is ever increasing.

A stochastic process is a family of random variables $\left{X_{t}\right}$, where $t$ takes values in the index set $T$ (sometimes called a parameter set or a time set).
The values of $X_{t}$ are called the state space and will be denoted by $S$.
If $T$ is countable then the stochastic process is called a stochastic sequence (or discrete parameter stochastic process). If $S$ is countable then the stochastic process is called a discrete state (space) process.

If $S$ is a subset of the real line the stochastic process is called a real valued process.
If $T$ takes continuously uncountable number of values like $(0, \infty)$ or $(-\infty, \infty)$ the stochastic process is called a continuous time process. To emphasize its dependence on $t$ and sample point $w$, we shall denote the stochastic process by $X(t, w), t \in T, w \in \Omega$ i.e. for each $w \in \Omega, X_{t}=X(t$,
$w)$ is a function of $t$.
This graph is known as the “typical sample function” or “realization of the stochastic process” $X(t, w)$.

数学代写|随机过程统计代写Stochastic process statistics代考|Different Types of Stochastic Processes

Following are the most important types of stochastic processes we come across:

  1. Independent stochastic sequence (Discrete time process)
    $T={1,2,3, \ldots}$ and $\left{X_{t}, t \in T\right}$ are independent random variables.
  2. Renewal process (Discrete time process)
    Here $T={0,1,2,3, \ldots], S=[0, \infty]$.
    If $X_{n}$ are i.i.d. non-negative random variables and $S_{n}=X_{1}+\ldots+X_{n}$ then $\left{S_{n}\right}$ forms a discrete time (renewal process).
  3. Independent increment process (Continuous time process)
    $T=\left{t_{0}, \infty\right}$, where $t_{0}$ be any real number (+ or $-$ ). For every
    $$
    t_{0}<t_{1}<\ldots<t_{n}, t_{i} \in T, i=1,2, \ldots, n
    $$
    if $X_{t_{0}}, X_{t_{1}}-X_{t_{0}}, X_{t_{2}}-X_{t_{1}}, \ldots, X_{t_{n}}-X_{t_{n-1}}$ are independent for all possible choices of $(1.1)$, then the stochastic process $\left{X_{r}, t \in T\right}$ is called independent increment stochastic process.
  4. Markov process
    $$
    \text { If } \begin{aligned}
    P\left[X_{i_{n+1}} \in A \mid X_{i_{n}}=a_{n}\right.&\left., X_{t_{n-1}}=a_{n-1}, \ldots, X_{t_{0}}=a_{0}\right] \
    &=P\left[X_{t_{n+1}} \in A \mid X_{t_{n}}=a_{n}\right] \text { holds for all choices of } \
    t_{0}<t_{1}<t_{2} &<\ldots<t_{n+1}, t_{i} \in T \cdot i=0,1,2, \ldots, n+1
    \end{aligned}
    $$
    and $A \in B$, the Borel field of the state space $S$, then $\left{X_{t}, t \in T\right}$ is called a Markov process.
  5. Martingale or fair game process
    If
    $$
    E\left[X_{t_{n+1}} \mid X_{t_{n}}=a_{n}, X_{t_{n-1}}=a_{n-1}, \ldots, X_{t_{0}}=a_{0}\right]=a_{n}
    $$
    i.e. $E\left[X_{t_{n+1}} \mid X_{t_{n}}, \ldots, X_{t_{0}}\right]=X_{t_{n}}$ a.s. for all choices of the partition (1.1), then $\left{X_{t}, t \in T\right}$ is called a Martingale process.
  6. Stationary process
    If the joint distribution of $\left(X_{t_{1}+t_{h}}, \ldots, X_{t_{n}+h}\right)$ are the same for all $h>0$ and
    $$
    t_{1}<t_{2}<\ldots<t_{n}, t_{i} \in T, t_{i}+h \in T
    $$
    then $\left{X_{t}, t \in T\right}$ is called a stationary process (strictly stationary process).

数学代写|随机过程统计代写Stochastic process statistics代考|An Introduction to Stationary Processes

A stochastic process $\left{X_{t}, t \in T\right}$ with $E X_{t}^{2}<\infty$ for all $t \in T$ is called covariance stationary or stationary in the wide-sense or weakly stationary if its covariance

function $C_{s, t}=E\left(X_{1} X_{s}\right)$ depends only on the difference $|t-s|$ for all $t, s \in T$. Nute that in our definition we have taken a zero mean stochastic process.
Examples of stationary processes
(a) Electrical pulses in communication theory are often postulated to describe a stationary process. Of course, in any physical system there is a transient period at the beginning of a signal. Since typically this has a short duration compared to the signal length, a stationary model may be appropriate. In electrical communication theory, often both the electrical potential and the current are represented as complex variables. Here we may encounter complex-valued stationary processes.
(b) The spatial and/or planar distributions of stars of galaxies, plants and animals, are often stationary. Time parameter set $T$ might be Euclidean space, the surface of a sphere or the plane.

A stationary distribution may be postulated for the height of a wave and $T$ is taken to be a set of longitudes and latitudes, again two dimensional.
(c) Economic time series, such as unemployment, gross national product, national income etc., are often assumed to correspond to a stationary process, at least after some correction for long-term growth has been made.

数学代写|随机过程统计代写Stochastic process statistics代考|STAT4061

随机过程统计代考

数学代写|随机过程统计代写Stochastic process statistics代考|Notion of Stochastic Processes

粗略地说,随时间变化的随机现象的数学描述是一个随机过程。自上个世纪以来,人们更多地认识到随机(或非确定性)模型在许多情况下比确定性模型更现实。在不同时间点进行的观察,而不是在固定时间段进行的观察开始引起科学家的注意。物理学家和通信工程师在动态非决定论的发展中发挥了主导作用。物理和生命科学中发生的许多现象不仅被研究为随机现象,而且被研究为随时间或空间变化的现象。在社会科学、经济和管理科学等其他领域也有类似的考虑。

随机过程是一系列随机变量\left{X_{t}\right}\left{X_{t}\right}, 在哪里吨取索引集中的值吨(有时称为参数集或时间集)。
的价值观X吨称为状态空间,记为小号.
如果吨是可数的,则随机过程称为随机序列(或离散参数随机过程)。如果小号是可数的,则该随机过程称为离散状态(空间)过程。

如果小号是实线的子集,随机过程称为实值过程。
如果吨连续获取无数个值,例如(0,∞)或者(−∞,∞)随机过程称为连续时间过程。强调它的依赖吨和采样点在,我们将随机过程表示为X(吨,在),吨∈吨,在∈Ω即对于每个在∈Ω,X吨=X(吨,
在)是一个函数吨.
该图被称为“典型样本函数”或“随机过程的实现”X(吨,在).

数学代写|随机过程统计代写Stochastic process statistics代考|Different Types of Stochastic Processes

以下是我们遇到的最重要的随机过程类型:

  1. 独立随机序列(离散时间过程)
    吨=1,2,3,…和\left{X_{t}, t \in T\right}\left{X_{t}, t \in T\right}是独立的随机变量。
  2. 更新过程(离散时间过程)
    这里 $T={0,1,2,3, \ldots], S=[0, \infty].我FX_{n}一个r和一世.一世.d.n○n−n和G一个吨一世在和r一个nd○米在一个r一世一个bl和s一个ndS_{n}=X_{1}+\ldots+X_{n}吨H和n\left{S_{n}\right}$ 形成一个离散时间(更新过程)。
  3. 独立增量过程(Continuous time process)
    T=\left{t_{0}, \infty\right}T=\left{t_{0}, \infty\right}, 在哪里吨0是任何实数(+ 或−)。对于每一个
    吨0<吨1<…<吨n,吨一世∈吨,一世=1,2,…,n
    如果X吨0,X吨1−X吨0,X吨2−X吨1,…,X吨n−X吨n−1对于所有可能的选择都是独立的(1.1),然后是随机过程\left{X_{r}, t \in T\right}\left{X_{r}, t \in T\right}称为独立增量随机过程。
  4. 马尔科夫过程
     如果 磷[X一世n+1∈一个∣X一世n=一个n,X吨n−1=一个n−1,…,X吨0=一个0] =磷[X吨n+1∈一个∣X吨n=一个n] 适用于所有选择  吨0<吨1<吨2<…<吨n+1,吨一世∈吨⋅一世=0,1,2,…,n+1
    和一个∈乙, 状态空间的 Borel 场小号, 然后\left{X_{t}, t \in T\right}\left{X_{t}, t \in T\right}称为马尔科夫过程。
  5. 鞅或公平博弈过程
    If
    和[X吨n+1∣X吨n=一个n,X吨n−1=一个n−1,…,X吨0=一个0]=一个n
    IE和[X吨n+1∣X吨n,…,X吨0]=X吨n至于分区(1.1)的所有选择,那么\left{X_{t}, t \in T\right}\left{X_{t}, t \in T\right}称为鞅过程。
  6. 平稳过程
    如果联合分布(X吨1+吨H,…,X吨n+H)所有人都一样H>0和
    吨1<吨2<…<吨n,吨一世∈吨,吨一世+H∈吨
    然后\left{X_{t}, t \in T\right}\left{X_{t}, t \in T\right}称为平稳过程(strictly平稳过程)。

数学代写|随机过程统计代写Stochastic process statistics代考|An Introduction to Stationary Processes

随机过程\left{X_{t}, t \in T\right}\left{X_{t}, t \in T\right}和和X吨2<∞对所有人吨∈吨称为协方差平稳或广义上的平稳或弱平稳,如果它的协方差

功能Cs,吨=和(X1Xs)仅取决于差异|吨−s|对所有人吨,s∈吨. 请注意,在我们的定义中,我们采用了零均值随机过程。
静止过程的例子
(a) 通信理论中的电脉冲通常被假定为描述静止过程。当然,在任何物理系统中,信号开始时都有一个瞬态周期。由于与信号长度相比,这通常具有较短的持续时间,因此固定模型可能是合适的。在电通信理论中,通常电势和电流都表示为复变量。在这里,我们可能会遇到复值平稳过程。
(b) 星系、植物和动物的恒星的空间和/或平面分布通常是静止的。时间参数集吨可能是欧几里得空间、球面或平面。

可以假设波浪的高度和吨被视为一组经度和纬度,也是二维的。
(c) 经济时间序列,例如失业、国民生产总值、国民收入等,通常被假定为对应于一个平稳的过程,至少在对长期增长进行了一些修正之后是这样。

数学代写|随机过程统计代写Stochastic process statistics代考 请认准statistics-lab™

统计代写请认准statistics-lab™. statistics-lab™为您的留学生涯保驾护航。

金融工程代写

金融工程是使用数学技术来解决金融问题。金融工程使用计算机科学、统计学、经济学和应用数学领域的工具和知识来解决当前的金融问题,以及设计新的和创新的金融产品。

非参数统计代写

非参数统计指的是一种统计方法,其中不假设数据来自于由少数参数决定的规定模型;这种模型的例子包括正态分布模型和线性回归模型。

广义线性模型代考

广义线性模型(GLM)归属统计学领域,是一种应用灵活的线性回归模型。该模型允许因变量的偏差分布有除了正态分布之外的其它分布。

术语 广义线性模型(GLM)通常是指给定连续和/或分类预测因素的连续响应变量的常规线性回归模型。它包括多元线性回归,以及方差分析和方差分析(仅含固定效应)。

有限元方法代写

有限元方法(FEM)是一种流行的方法,用于数值解决工程和数学建模中出现的微分方程。典型的问题领域包括结构分析、传热、流体流动、质量运输和电磁势等传统领域。

有限元是一种通用的数值方法,用于解决两个或三个空间变量的偏微分方程(即一些边界值问题)。为了解决一个问题,有限元将一个大系统细分为更小、更简单的部分,称为有限元。这是通过在空间维度上的特定空间离散化来实现的,它是通过构建对象的网格来实现的:用于求解的数值域,它有有限数量的点。边界值问题的有限元方法表述最终导致一个代数方程组。该方法在域上对未知函数进行逼近。[1] 然后将模拟这些有限元的简单方程组合成一个更大的方程系统,以模拟整个问题。然后,有限元通过变化微积分使相关的误差函数最小化来逼近一个解决方案。

tatistics-lab作为专业的留学生服务机构,多年来已为美国、英国、加拿大、澳洲等留学热门地的学生提供专业的学术服务,包括但不限于Essay代写,Assignment代写,Dissertation代写,Report代写,小组作业代写,Proposal代写,Paper代写,Presentation代写,计算机作业代写,论文修改和润色,网课代做,exam代考等等。写作范围涵盖高中,本科,研究生等海外留学全阶段,辐射金融,经济学,会计学,审计学,管理学等全球99%专业科目。写作团队既有专业英语母语作者,也有海外名校硕博留学生,每位写作老师都拥有过硬的语言能力,专业的学科背景和学术写作经验。我们承诺100%原创,100%专业,100%准时,100%满意。

随机分析代写


随机微积分是数学的一个分支,对随机过程进行操作。它允许为随机过程的积分定义一个关于随机过程的一致的积分理论。这个领域是由日本数学家伊藤清在第二次世界大战期间创建并开始的。

时间序列分析代写

随机过程,是依赖于参数的一组随机变量的全体,参数通常是时间。 随机变量是随机现象的数量表现,其时间序列是一组按照时间发生先后顺序进行排列的数据点序列。通常一组时间序列的时间间隔为一恒定值(如1秒,5分钟,12小时,7天,1年),因此时间序列可以作为离散时间数据进行分析处理。研究时间序列数据的意义在于现实中,往往需要研究某个事物其随时间发展变化的规律。这就需要通过研究该事物过去发展的历史记录,以得到其自身发展的规律。

回归分析代写

多元回归分析渐进(Multiple Regression Analysis Asymptotics)属于计量经济学领域,主要是一种数学上的统计分析方法,可以分析复杂情况下各影响因素的数学关系,在自然科学、社会和经济学等多个领域内应用广泛。

MATLAB代写

MATLAB 是一种用于技术计算的高性能语言。它将计算、可视化和编程集成在一个易于使用的环境中,其中问题和解决方案以熟悉的数学符号表示。典型用途包括:数学和计算算法开发建模、仿真和原型制作数据分析、探索和可视化科学和工程图形应用程序开发,包括图形用户界面构建MATLAB 是一个交互式系统,其基本数据元素是一个不需要维度的数组。这使您可以解决许多技术计算问题,尤其是那些具有矩阵和向量公式的问题,而只需用 C 或 Fortran 等标量非交互式语言编写程序所需的时间的一小部分。MATLAB 名称代表矩阵实验室。MATLAB 最初的编写目的是提供对由 LINPACK 和 EISPACK 项目开发的矩阵软件的轻松访问,这两个项目共同代表了矩阵计算软件的最新技术。MATLAB 经过多年的发展,得到了许多用户的投入。在大学环境中,它是数学、工程和科学入门和高级课程的标准教学工具。在工业领域,MATLAB 是高效研究、开发和分析的首选工具。MATLAB 具有一系列称为工具箱的特定于应用程序的解决方案。对于大多数 MATLAB 用户来说非常重要,工具箱允许您学习应用专业技术。工具箱是 MATLAB 函数(M 文件)的综合集合,可扩展 MATLAB 环境以解决特定类别的问题。可用工具箱的领域包括信号处理、控制系统、神经网络、模糊逻辑、小波、仿真等。

R语言代写问卷设计与分析代写
PYTHON代写回归分析与线性模型代写
MATLAB代写方差分析与试验设计代写
STATA代写机器学习/统计学习代写
SPSS代写计量经济学代写
EVIEWS代写时间序列分析代写
EXCEL代写深度学习代写
SQL代写各种数据建模与可视化代写

数学代写|随机过程统计代写Stochastic process statistics代考|MATH3801

如果你也在 怎样代写随机过程统计Stochastic process statistics这个学科遇到相关的难题,请随时右上角联系我们的24/7代写客服。

随机过程 用于表示在时间上发展的统计现象以及在处理这些现象时出现的理论模型,由于这些现象在许多领域都会遇到,因此这篇文章具有广泛的实际意义。

statistics-lab™ 为您的留学生涯保驾护航 在代写随机过程统计Stochastic process statistics方面已经树立了自己的口碑, 保证靠谱, 高质且原创的统计Statistics代写服务。我们的专家在代写随机过程统计Stochastic process statistics代写方面经验极为丰富,各种代写随机过程统计Stochastic process statistics相关的作业也就用不着说。

我们提供的随机过程统计Stochastic process statistics及其相关学科的代写,服务范围广, 其中包括但不限于:

  • Statistical Inference 统计推断
  • Statistical Computing 统计计算
  • Advanced Probability Theory 高等概率论
  • Advanced Mathematical Statistics 高等数理统计学
  • (Generalized) Linear Models 广义线性模型
  • Statistical Machine Learning 统计机器学习
  • Longitudinal Data Analysis 纵向数据分析
  • Foundations of Data Science 数据科学基础
数学代写|随机过程统计代写Stochastic process statistics代考|MATH3801

数学代写|随机过程统计代写Stochastic process statistics代考|Importance of Software for Process Analysis

In process engineering, the simulation, design, and optimization of a chemical process plant, which comprises several processing units interconnected by process streams, are the core activities. These tasks require material and energy balancing, equipment sizing, and costing calculation. A computer package that can accomplish these duties is known as a computer-aided process design package or simply a process simulator (also known as process flowsheeting package, flowsheet simulator, or flowsheeting software). The capabilities of a process simulator include an accurate description of physical properties of pure components and complex mixtures, rigorous models for unit operations, as well as numerical techniques for solving large systems of algebraic and differential equations. By a process simulator, it is possible to obtain a comprehensive computer image of a running process, which is a valuable tool in understanding the operation of a complex chemical plant and on this basis can serve for continuously improving the process or for developing new processes.

The purpose of simulation is to model and predict the performance of a process. It involves decomposition of the process into its constituent units for individual study of performance. The process characteristics (e.g., flow rates, compositions, temperatures, pressures, properties, and equipment sizes) are predicted using analysis techniques, which include mathematical models, empirical correlations, and computer-aided process simulation tools (e.g., Aspen Plus). In addition, process analysis may involve the use of experimental methods to predict and validate performance. Therefore, in process simulation, the process inputs and the flowsheet are given, and we are required to predict process outputs (Figure 4.1). This book focuses on Aspen Plus. It is a computer-aided software that uses the underlying physical relationships (e.g., material and energy balances, thermodynamic equilibrium, and rate equations) to predict process performance (e.g., stream properties, operating conditions, and equipment sizes).
There are several advantages of computer-aided simulation:

  1. It allows the designer to quickly test the performance of synthesized process flowsheets and provide feedback to the process synthesis activities.
  2. It can be coordinated with process synthesis to develop optimum integrated designs.
  3. It minimizes experimental and scale-up efforts.
  4. It explores process flexibility and sensitivity by answering “what-if” questions.
  5. It quantitatively models the process and sheds insights on process performance.
  6. Following are the important issues to remember before venturing into the exciting world of computer-aided simulation:
  7. Do not implicitly trust the results of any simulation tool.
  8. Calculated results are only as good as the input you give the simulator.
  9. Always convince yourself that the obtained results make physical sense, otherwise you will never be able to convince someone else of the merits of your work.

数学代写|随机过程统计代写Stochastic process statistics代考|Characteristics of the Process Simulator Aspen Plus

The process simulation market underwent severe transformations in the 1985-1995 decade. Relatively few systems have survived; they are CHEMCAD, Aspen Plus, Aspen HYSYS, PRO/II, ProSimPlus, SuperPro Designer, and gPROMS. Nowadays, most of the current process simulators are developed following an object-oriented approach using languages such as $\mathrm{C}_{+}+$or Java. This shift in paradigm, from procedural to object-oriented, has no doubt benefited and will continue to benefit the process engineering community immensely.

Aspen Plus is designed for the simulation of steady-state processes; especially those that are computationally laborious to analyze by hand calculations, such as processes involving recycle streams, nonideal phase or chemical

equilibria, and adiabatic operations. It is ideally suited to provide answers on “what-if” type of questions on process design and optimization.

Fundamental to improving performance of the plant is an accurate representation of the basic processes. Companies require a solution that enables them to model their processes to develop insights to improve designs and optimize performance. Aspen Plus provides the solution to meet this requirement, solving the critical engineering and operating problems that arise throughout the life cycle of a chemical process.

Aspen Plus predicts process behavior using engineering relationships, such as mass and energy balances, phase and chemical equilibria, and reaction kinetics. With reliable physical properties, thermodynamic data, realistic operating conditions, and rigorous equipment models, engineers are able to simulate actual plant behaviors. Applications include the following:

  • Improving engineering productivity and reducing costs
  • Reducing energy consumption and greenhouse gas emissions
  • Enhancing product yields and quality
  • Minimizing capital and operating costs
  • Optimizing designs for large-scale integrated chemical plants
  • Optimizing plant operations

数学代写|随机过程统计代写Stochastic process statistics代考|Sequential Modular Simulation

The sequential modular approach is based on the concept of modularity, which extends the chemical engineering concept of unit operation to a “unit calculation” of the computer code (i.e., subroutine) responsible for the calculations of an equipment. This method is similar in principle to the traditional method of hand calculation of unit operations. The equations for each equipment unit are grouped together in a subroutine or module. Thus, each module calculates the output streams for the given input streams and parameters for that equipment, irrespective of the source of input information or the sink of output information. In the equation-oriented type, the complete model of the plant is expressed in the form of one large sparse system of nonlinear algebraic equations that is simultaneously solved for all the unknowns. This approach combines the modularizing of the equations related to specific equipment with the efficient solution algorithms for the simultaneous equation-solving technique. For each unit, an additional module is written, which approximately relates each output value by a linear combination of all input values. Accordingly, rigorous models are used at units’ level, which are solved sequentially, whereas linear models are used at flowsheet level, solved globally. The linear models are updated based on results obtained with rigorous models (Martin-Martin, 2015).
Following are the basic components of a simulation package:

  1. Component data bank
  2. Thermodynamic property prediction methods
  3. Flowsheet builder (graphical user interface)
  4. Unit module library
  5. Numerical routines
  6. Data output generator
  7. Executive program (flowsheet solver)
    Modular process simulators are very robust solving each unit operation with numerical methods tailored to the specific characteristics to each one of these units. These include from specific inside-out algorithms to “flash” a material stream going through detailed methods for reactors and heat exchangers until complex methods for distillation. However, one drawback of the approach is that some unit operations introduce numerical noise (this is also simulator and unit dependent). In other words, if we solve the same problem starting from different initial points, we will obtain, for some variables, slightly different values. The difference can be in the second or third decimal point, which is not significant from a simulation point of view but is a really large error if we try to estimate a derivative (derivative information is not provided by the simulator, although some unit operations internally use it to solve the module). This problem is magnified by information loops that could act as “error accumulators.”
数学代写|随机过程统计代写Stochastic process statistics代考|MATH3801

随机过程统计代考

数学代写|随机过程统计代写Stochastic process statistics代考|Importance of Software for Process Analysis
在过程工程中,化学过程工厂的模拟、设计和优化是核心活动,其中包括多个通过过程流相互连接的处理单元。这些任务需要材料和能量平衡、设备选型和成本计算。可以完成这些任务的计算机软件包称为计算机辅助工艺设计软件包或简称为工艺模拟器(也称为工艺流程图软件包、流程图模拟器或流程图软件)。过程模拟器的功能包括准确描述纯组分和复杂混合物的物理特性、单元操作的严格模型,以及求解大型代数和微分方程系统的数值技术。通过过程模拟器,

模拟的目的是对过程的性能进行建模和预测。它涉及将过程分解为其组成单元,以进行单独的绩效研究。使用包括数学模型、经验相关性和计算机辅助过程模拟工具(例如 Aspen Plus)在内的分析技术预测过程特性(例如,流速、成分、温度、压力、性质和设备尺寸)。此外,过程分析可能涉及使用实验方法来预测和验证性能。因此,在过程模拟中,给出了过程输入和流程图,我们需要预测过程输出(图 4.1)。本书重点介绍 Aspen Plus。它是一种计算机辅助软件,它使用底层的物理关系(例如,
计算机辅助模拟有几个优点:

它允许设计人员快速测试合成工艺流程图的性能,并为工艺合成活动提供反馈。
它可以与工艺合成相协调,以开发最佳的集成设计。
它最大限度地减少了实验和扩大规模。
它通过回答“假设”问题来探索流程的灵活性和敏感性。
它对流程进行定量建模,并揭示流程绩效。
以下是在进入令人兴奋的计算机辅助模拟世界之前要记住的重要问题:
不要暗中相信任何模拟工具的结果。
计算结果与您给模拟器的输入一样好。
始终说服自己获得的结果具有物理意义,否则您将永远无法说服其他人您工作的优点。
数学代写|随机过程统计代写Stochastic process statistics代考|Characteristics of the Process Simulator Aspen Plus
过程模拟市场在 1985-1995 十年期间经历了剧烈的转变。幸存下来的系统相对较少;它们是 CHEMCAD、Aspen Plus、Aspen HYSYS、PRO/II、ProSimPlus、SuperPro Designer 和 gPROMS。如今,大多数当前的过程模拟器都是按照面向对象的方法开发的,使用诸如或Java。这种范式的转变,从面向过程到面向对象,无疑已经并将继续极大地造福于过程工程社区。

Aspen Plus 专为模拟稳态过程而设计;尤其是那些通过手工计算难以计算的分析,例如涉及循环流、非理想相或化学的过程

平衡和绝热操作。它非常适合提供有关流程设计和优化的“假设”类型问题的答案。

提高工厂绩效的基础是基本流程的准确表示。公司需要一种解决方案,使他们能够对其流程进行建模,以开发洞察力以改进设计和优化性能。Aspen Plus 提供了满足这一要求的解决方案,解决了整个化学过程生命周期中出现的关键工程和操作问题。

Aspen Plus 使用工程关系预测过程行为,例如质量和能量平衡、相和化学平衡以及反应动力学。凭借可靠的物理特性、热力学数据、真实的运行条件和严格的设备模型,工程师能够模拟实际的工厂行为。应用包括以下内容:

提高工程生产力并降低成本
减少能源消耗和温室气体排放
提高产品产量和质量
最大限度地降低资本和运营成本
大型综合化工厂的优化设计
优化工厂运营
数学代写|随机过程统计代写Stochastic process statistics代考|Sequential Modular Simulation
顺序模块化方法基于模块化概念,它将单元操作的化学工程概念扩展到负责设备计算的计算机代码(即子程序)的“单元计算”。这种方法在原理上与传统的单位操作手工计算方法相似。每个设备单元的方程组合在一个子程序或模块中。因此,每个模块计算给定输入流的输出流和该设备的参数,而不考虑输入信息的来源或输出信息的接收器。在面向方程的类型中,被控对象的完整模型以一个大型稀疏非线性代数方程系统的形式表示,该系统同时求解所有未知数。这种方法将与特定设备相关的方程的模块化与联立方程求解技术的有效求解算法相结合。对于每个单元,都会编写一个附加模块,该模块通过所有输入值的线性组合近似地关联每个输出值。因此,严格模型用于单元级,按顺序求解,而线性模型用于流程图级,全局求解。线性模型根据使用严格模型获得的结果进行更新(Martin-Martin,2015)。因此,严格模型用于单元级,按顺序求解,而线性模型用于流程图级,全局求解。线性模型根据使用严格模型获得的结果进行更新(Martin-Martin,2015)。因此,严格模型用于单元级,按顺序求解,而线性模型用于流程图级,全局求解。线性模型根据使用严格模型获得的结果进行更新(Martin-Martin,2015)。
以下是模拟包的基本组件:

组件数据库
热力学性质预测方法
流程图生成器(图形用户界面)
单元模块库
数值例程
数据输出发生器
执行程序(流程图求解器)
模块化过程模拟器非常强大,可以使用针对每个单元的特定特征量身定制的数值方法来解决每个单元操作。这些包括从特定的由内而外的算法到“闪蒸”物料流,通过反应器和热交换器的详细方法,直到复杂的蒸馏方法。然而,该方法的一个缺点是某些单元操作会引入数值噪声(这也是模拟器和单元相关的)。换句话说,如果我们从不同的初始点开始解决同一个问题,对于某些变量,我们将获得稍微不同的值。差异可以在第二个或第三个小数点,从模拟的角度来看,这并不重要,但如果我们尝试估计导数,则会产生非常大的误差(模拟器不提供导数信息,尽管一些单元操作在内部使用它来求解模块)。这个问题被可能充当“错误累加器”的信息循环放大了。

数学代写|随机过程统计代写Stochastic process statistics代考 请认准statistics-lab™

统计代写请认准statistics-lab™. statistics-lab™为您的留学生涯保驾护航。

金融工程代写

金融工程是使用数学技术来解决金融问题。金融工程使用计算机科学、统计学、经济学和应用数学领域的工具和知识来解决当前的金融问题,以及设计新的和创新的金融产品。

非参数统计代写

非参数统计指的是一种统计方法,其中不假设数据来自于由少数参数决定的规定模型;这种模型的例子包括正态分布模型和线性回归模型。

广义线性模型代考

广义线性模型(GLM)归属统计学领域,是一种应用灵活的线性回归模型。该模型允许因变量的偏差分布有除了正态分布之外的其它分布。

术语 广义线性模型(GLM)通常是指给定连续和/或分类预测因素的连续响应变量的常规线性回归模型。它包括多元线性回归,以及方差分析和方差分析(仅含固定效应)。

有限元方法代写

有限元方法(FEM)是一种流行的方法,用于数值解决工程和数学建模中出现的微分方程。典型的问题领域包括结构分析、传热、流体流动、质量运输和电磁势等传统领域。

有限元是一种通用的数值方法,用于解决两个或三个空间变量的偏微分方程(即一些边界值问题)。为了解决一个问题,有限元将一个大系统细分为更小、更简单的部分,称为有限元。这是通过在空间维度上的特定空间离散化来实现的,它是通过构建对象的网格来实现的:用于求解的数值域,它有有限数量的点。边界值问题的有限元方法表述最终导致一个代数方程组。该方法在域上对未知函数进行逼近。[1] 然后将模拟这些有限元的简单方程组合成一个更大的方程系统,以模拟整个问题。然后,有限元通过变化微积分使相关的误差函数最小化来逼近一个解决方案。

tatistics-lab作为专业的留学生服务机构,多年来已为美国、英国、加拿大、澳洲等留学热门地的学生提供专业的学术服务,包括但不限于Essay代写,Assignment代写,Dissertation代写,Report代写,小组作业代写,Proposal代写,Paper代写,Presentation代写,计算机作业代写,论文修改和润色,网课代做,exam代考等等。写作范围涵盖高中,本科,研究生等海外留学全阶段,辐射金融,经济学,会计学,审计学,管理学等全球99%专业科目。写作团队既有专业英语母语作者,也有海外名校硕博留学生,每位写作老师都拥有过硬的语言能力,专业的学科背景和学术写作经验。我们承诺100%原创,100%专业,100%准时,100%满意。

随机分析代写


随机微积分是数学的一个分支,对随机过程进行操作。它允许为随机过程的积分定义一个关于随机过程的一致的积分理论。这个领域是由日本数学家伊藤清在第二次世界大战期间创建并开始的。

时间序列分析代写

随机过程,是依赖于参数的一组随机变量的全体,参数通常是时间。 随机变量是随机现象的数量表现,其时间序列是一组按照时间发生先后顺序进行排列的数据点序列。通常一组时间序列的时间间隔为一恒定值(如1秒,5分钟,12小时,7天,1年),因此时间序列可以作为离散时间数据进行分析处理。研究时间序列数据的意义在于现实中,往往需要研究某个事物其随时间发展变化的规律。这就需要通过研究该事物过去发展的历史记录,以得到其自身发展的规律。

回归分析代写

多元回归分析渐进(Multiple Regression Analysis Asymptotics)属于计量经济学领域,主要是一种数学上的统计分析方法,可以分析复杂情况下各影响因素的数学关系,在自然科学、社会和经济学等多个领域内应用广泛。

MATLAB代写

MATLAB 是一种用于技术计算的高性能语言。它将计算、可视化和编程集成在一个易于使用的环境中,其中问题和解决方案以熟悉的数学符号表示。典型用途包括:数学和计算算法开发建模、仿真和原型制作数据分析、探索和可视化科学和工程图形应用程序开发,包括图形用户界面构建MATLAB 是一个交互式系统,其基本数据元素是一个不需要维度的数组。这使您可以解决许多技术计算问题,尤其是那些具有矩阵和向量公式的问题,而只需用 C 或 Fortran 等标量非交互式语言编写程序所需的时间的一小部分。MATLAB 名称代表矩阵实验室。MATLAB 最初的编写目的是提供对由 LINPACK 和 EISPACK 项目开发的矩阵软件的轻松访问,这两个项目共同代表了矩阵计算软件的最新技术。MATLAB 经过多年的发展,得到了许多用户的投入。在大学环境中,它是数学、工程和科学入门和高级课程的标准教学工具。在工业领域,MATLAB 是高效研究、开发和分析的首选工具。MATLAB 具有一系列称为工具箱的特定于应用程序的解决方案。对于大多数 MATLAB 用户来说非常重要,工具箱允许您学习应用专业技术。工具箱是 MATLAB 函数(M 文件)的综合集合,可扩展 MATLAB 环境以解决特定类别的问题。可用工具箱的领域包括信号处理、控制系统、神经网络、模糊逻辑、小波、仿真等。

R语言代写问卷设计与分析代写
PYTHON代写回归分析与线性模型代写
MATLAB代写方差分析与试验设计代写
STATA代写机器学习/统计学习代写
SPSS代写计量经济学代写
EVIEWS代写时间序列分析代写
EXCEL代写深度学习代写
SQL代写各种数据建模与可视化代写

数学代写|随机过程统计代写Stochastic process statistics代考|STAT7004

如果你也在 怎样代写随机过程统计Stochastic process statistics这个学科遇到相关的难题,请随时右上角联系我们的24/7代写客服。

随机过程 用于表示在时间上发展的统计现象以及在处理这些现象时出现的理论模型,由于这些现象在许多领域都会遇到,因此这篇文章具有广泛的实际意义。

statistics-lab™ 为您的留学生涯保驾护航 在代写随机过程统计Stochastic process statistics方面已经树立了自己的口碑, 保证靠谱, 高质且原创的统计Statistics代写服务。我们的专家在代写随机过程统计Stochastic process statistics代写方面经验极为丰富,各种代写随机过程统计Stochastic process statistics相关的作业也就用不着说。

我们提供的随机过程统计Stochastic process statistics及其相关学科的代写,服务范围广, 其中包括但不限于:

  • Statistical Inference 统计推断
  • Statistical Computing 统计计算
  • Advanced Probability Theory 高等概率论
  • Advanced Mathematical Statistics 高等数理统计学
  • (Generalized) Linear Models 广义线性模型
  • Statistical Machine Learning 统计机器学习
  • Longitudinal Data Analysis 纵向数据分析
  • Foundations of Data Science 数据科学基础
数学代写|随机过程统计代写Stochastic process statistics代考|STAT7004

数学代写|随机过程统计代写Stochastic process statistics代考|Simulated Annealing

Kirkpatrick et al. (1983) established that there is a similarity between the behavior of a system reaching thermal equilibrium and the performance of an optimization procedure. This was considered the basis for the creation of the simulated annealing approach, which emulates the phenomena of annealing in solids. In the annealing procedure, a solid is first heated at high temperature. At this stage, the energy of the system is quite high, and the atoms in the solid are randomly distributed. Then, the temperature is slowly reduced until a new equilibrium is reached. This procedure continues until the atoms are ordered in a crystalline structure, where the energy of the system is at its minimum. The solid treated with annealing is quite resistant. However, if the temperature was not gradually reduced, a thermal shock is induced, and the solid becomes fragile. In the simulated annealing method, a simulated temperature is used to control the algorithm. As in the physical phenomena, this temperature must be high to cause a random behavior. Once the initial simulated temperature is selected, an initial solution $\bar{x}$ is proposed, and the value of the objective function $f(\bar{x})$ is computed. Then, a second solution $\bar{x}^{\prime}$ is proposed, computing the value of $f\left(\bar{x}^{\prime}\right)$. The two solutions are compared and, if $f\left(\bar{x}^{\prime}\right)$ is better than $\mathrm{f}(\bar{x})$ (i.e., if, for a minimization, $\mathrm{f}\left(\bar{x}^{\prime}\right) \geq \mathrm{f}(\bar{x})$ ), then $\bar{x}^{\prime}$ is selected as the new solution. If $\mathrm{f}\left(\bar{x}^{\prime}\right)$ is worse than $\mathrm{f}(\bar{x})$, it is not immediately discarded. Instead, a probability of selection is computed using the Metropolis formula (Metropolis et al., 1953):
$$
P(\Delta)=\exp \left(-\frac{\Delta}{T}\right)
$$
where
$$
\Delta=\mathrm{f}\left(\bar{x}^{\prime}\right)-\mathrm{f}(\bar{x})
$$
If the probability of selection is higher than a random number $a$, then $\bar{x}^{\prime}$ is selected as the new solution. Otherwise, the method returns to the previous proposal, $\bar{x}$. This implies that, if a given solution is “bad,” it still has probabilities of being selected as a new solution. This is helpful to perform a search on all feasible regions, avoiding local optimum. The proposal and selection of new solutions continue until a stationary point is reached. Then, the temperature is decreased, and a new set of proposals is established. As the temperature decreases, the probability of selection is lower. Thus, when the algorithm advances, the “bad” solutions have less chances to be selected because the method is expected to be converging to the global optimum. The algorithm stops when the freezing temperature $\left(T_{\text {freeze }}\right)$ is reached, where the solution is stable and the same solution is selected among a certain number of proposals. Figure $3.6$ shows a graphical representation of the simulated annealing.

数学代写|随机过程统计代写Stochastic process statistics代考|Ant Colony Optimization

The ant colony optimization method was developed by Dorigo and Gambardella (1997) and is inspired by the behavior of ant colonies. In general, the ants indicate the way from their nest to a food source by depositing pheromones on the ground. When various ants are taking different paths, the path representing the shortest distance will have a higher concentration of pheromones, thus most of the ants will be attracted to follow that route and will increase the concentration of pheromones even more. In the end, all the ants nearby will follow the shortest path. The ant colony optimization method emulates this behavior. Two parameters are of importance to the algorithm: the pheromone value and the age of a solution. The algorithm starts with a randomly generated set of solutions. Then, a local search procedure starts, where a simulated ant selects a solution, in terms of the pheromone values. Then, in terms of the age of the current solution, a new solution is selected. If the fitness function of the new solution is better than that of the previous solution, the new solution is selected. Otherwise, the previous solution remains. In both cases, the values of age and pheromone are modified. Then, a global search takes place and the pheromone values are updated to consider the phenomena of pheromone evaporation. The procedure continues until the CC is achieved (Jayaraman et al., 2010). This method has been used for applications such as the optimization of project scheduling (Merkle et al., 2002), the optimization of water distribution systems (Maier et al., 2003), and the scheduling of batch pro-cesses (Jayaraman et al., 2010). Nevertheless, to the best of the authors’ knowlchemical processes.

数学代写|随机过程统计代写Stochastic process statistics代考|Particle Swarm Optimization

This optimization method is based on the social behavior of animal species, particularly that of human beings. The method was proposed by Kennedy and Eberhart (1995), and it is related to both artificial life and evolutionary programming. It has been developed from the observations and simulations of Reynolds (1987) not only about the movement patterns of flying animals (birds), land animals, and water animals (fishes), but also considering the abstractness that characterizes the human decision-making. In general, the algorithm functions with a set of entities known as particles. An initial population of particles is first generated, and a position and a velocity are assigned to each particle, where the position is given by a solution for the optimization problem. Then, the fitness function is evaluated for each position, and the best value is selected. Then, the position and the velocity of each particle are updated, which implies a movement of the particle in the direction of the best previous solution. The movements continue until a stop criterion is reached (Jarboui et al., 2010). This approach has been applied to the dynamic analysis of reactive systems (Ourique et al., 2002), the parameter estimation of a polypropylene reactor (Martinez Prata et al., 2009), and the optimal design of heat exchangers (Patel and Rao, 2010).

数学代写|随机过程统计代写Stochastic process statistics代考|STAT7004

随机过程统计代考

数学代写|随机过程统计代写Stochastic process statistics代考|Simulated Annealing

柯克帕特里克等人。(1983) 确定达到热平衡的系统行为与优化程序的性能之间存在相似性。这被认为是创建模拟退火方法的基础,该方法模拟了固体中的退火现象。在退火过程中,固体首先在高温下加热。在这个阶段,系统的能量相当高,固体中的原子是随机分布的。然后,温度慢慢降低,直到达到新的平衡。这个过程一直持续到原子排列成晶体结构,此时系统的能量最小。经退火处理的固体非常耐腐蚀。但是,如果温度没有逐渐降低,则会引发热冲击,固体变得脆弱。在模拟退火方法中,使用模拟温度来控制算法。与物理现象一样,该温度必须很高才能引起随机行为。一旦选择了初始模拟温度,初始解X¯提出,目标函数的值F(X¯)被计算。然后,第二个解决方案X¯′提出,计算的价值F(X¯′). 比较这两种解决方案,如果F(X¯′)好于F(X¯)(即,如果为了最小化,F(X¯′)≥F(X¯)), 然后X¯′被选为新解。如果F(X¯′)比F(X¯),它不会立即被丢弃。相反,选择概率是使用 Metropolis 公式计算的(Metropolis et al., 1953):

磷(Δ)=经验⁡(−Δ吨)
在哪里

Δ=F(X¯′)−F(X¯)
如果选择的概率高于随机数一个, 然后X¯′被选为新解。否则,该方法返回上一个提议,X¯. 这意味着,如果给定的解决方案是“坏的”,它仍然有可能被选为新的解决方案。这有助于在所有可行区域上执行搜索,避免局部最优。新解决方案的提议和选择会继续进行,直到达到一个固定点。然后,温度降低,并建立了一组新的建议。随着温度的降低,选择的概率会降低。因此,当算法推进时,“坏”解决方案被选择的机会更少,因为该方法预计会收敛到全局最优。当冻结温度时算法停止(吨冻结 )达到,其中解决方案是稳定的,并且在一定数量的提案中选择了相同的解决方案。数字3.6显示了模拟退火的图形表示。

数学代写|随机过程统计代写Stochastic process statistics代考|Ant Colony Optimization

蚁群优化方法由 Dorigo 和 Gambardella (1997) 开发,并受到蚁群行为的启发。一般来说,蚂蚁通过在地面上沉积信息素来指示从巢穴到食物来源的路径。当各种蚂蚁走不同的路径时,代表最短距离的路径的信息素浓度会更高,因此大多数蚂蚁会被吸引走这条路线,信息素的浓度会更高。最后,附近的所有蚂蚁都会走最短路径。蚁群优化方法模拟了这种行为。两个参数对算法很重要:信息素值和解的年龄。该算法从一组随机生成的解决方案开始。然后,本地搜索程序开始,其中模拟蚂蚁根据信息素值选择解决方案。然后,根据当前解决方案的年龄,选择一个新的解决方案。如果新解的适应度函数优于前一个解,则选择新解。否则,之前的解决方案仍然存在。在这两种情况下,年龄和信息素的值都会被修改。然后,进行全局搜索并更新信息素值以考虑信息素蒸发现象。该过程一直持续到达到 CC 为止(Jayaraman 等人,2010 年)。该方法已被用于项目调度优化 (Merkle et al., 2002)、配水系统优化 (Maier et al., 2003) 和批处理调度 (Jayaraman et al.) 等应用。 ., 2010)。尽管如此,

数学代写|随机过程统计代写Stochastic process statistics代考|Particle Swarm Optimization

这种优化方法是基于动物物种的社会行为,特别是人类的社会行为。该方法由 Kennedy 和 Eberhart (1995) 提出,它与人工生命和进化编程都有关。它是从 Reynolds (1987) 的观察和模拟发展而来的,不仅涉及飞行动物(鸟类)、陆地动物和水生动物(鱼类)的运动模式,还考虑了人类决策的抽象性。通常,该算法与一组称为粒子的实体一起运行。首先生成初始粒子群,并为每个粒子分配位置和速度,其中位置由优化问题的解给出。然后,评估每个位置的适应度函数,并选择最佳值。然后,更新每个粒子的位置和速度,这意味着粒子在最佳先前解的方向上移动。运动一直持续到达到停止标准(Jarboui et al., 2010)。这种方法已应用于反应系统的动态分析(Ourique 等人,2002)、聚丙烯反应器的参数估计(Martinez Prata 等人,2009)以及热交换器的优化设计(Patel 和 Rao, 2010)。

数学代写|随机过程统计代写Stochastic process statistics代考 请认准statistics-lab™

统计代写请认准statistics-lab™. statistics-lab™为您的留学生涯保驾护航。

金融工程代写

金融工程是使用数学技术来解决金融问题。金融工程使用计算机科学、统计学、经济学和应用数学领域的工具和知识来解决当前的金融问题,以及设计新的和创新的金融产品。

非参数统计代写

非参数统计指的是一种统计方法,其中不假设数据来自于由少数参数决定的规定模型;这种模型的例子包括正态分布模型和线性回归模型。

广义线性模型代考

广义线性模型(GLM)归属统计学领域,是一种应用灵活的线性回归模型。该模型允许因变量的偏差分布有除了正态分布之外的其它分布。

术语 广义线性模型(GLM)通常是指给定连续和/或分类预测因素的连续响应变量的常规线性回归模型。它包括多元线性回归,以及方差分析和方差分析(仅含固定效应)。

有限元方法代写

有限元方法(FEM)是一种流行的方法,用于数值解决工程和数学建模中出现的微分方程。典型的问题领域包括结构分析、传热、流体流动、质量运输和电磁势等传统领域。

有限元是一种通用的数值方法,用于解决两个或三个空间变量的偏微分方程(即一些边界值问题)。为了解决一个问题,有限元将一个大系统细分为更小、更简单的部分,称为有限元。这是通过在空间维度上的特定空间离散化来实现的,它是通过构建对象的网格来实现的:用于求解的数值域,它有有限数量的点。边界值问题的有限元方法表述最终导致一个代数方程组。该方法在域上对未知函数进行逼近。[1] 然后将模拟这些有限元的简单方程组合成一个更大的方程系统,以模拟整个问题。然后,有限元通过变化微积分使相关的误差函数最小化来逼近一个解决方案。

tatistics-lab作为专业的留学生服务机构,多年来已为美国、英国、加拿大、澳洲等留学热门地的学生提供专业的学术服务,包括但不限于Essay代写,Assignment代写,Dissertation代写,Report代写,小组作业代写,Proposal代写,Paper代写,Presentation代写,计算机作业代写,论文修改和润色,网课代做,exam代考等等。写作范围涵盖高中,本科,研究生等海外留学全阶段,辐射金融,经济学,会计学,审计学,管理学等全球99%专业科目。写作团队既有专业英语母语作者,也有海外名校硕博留学生,每位写作老师都拥有过硬的语言能力,专业的学科背景和学术写作经验。我们承诺100%原创,100%专业,100%准时,100%满意。

随机分析代写


随机微积分是数学的一个分支,对随机过程进行操作。它允许为随机过程的积分定义一个关于随机过程的一致的积分理论。这个领域是由日本数学家伊藤清在第二次世界大战期间创建并开始的。

时间序列分析代写

随机过程,是依赖于参数的一组随机变量的全体,参数通常是时间。 随机变量是随机现象的数量表现,其时间序列是一组按照时间发生先后顺序进行排列的数据点序列。通常一组时间序列的时间间隔为一恒定值(如1秒,5分钟,12小时,7天,1年),因此时间序列可以作为离散时间数据进行分析处理。研究时间序列数据的意义在于现实中,往往需要研究某个事物其随时间发展变化的规律。这就需要通过研究该事物过去发展的历史记录,以得到其自身发展的规律。

回归分析代写

多元回归分析渐进(Multiple Regression Analysis Asymptotics)属于计量经济学领域,主要是一种数学上的统计分析方法,可以分析复杂情况下各影响因素的数学关系,在自然科学、社会和经济学等多个领域内应用广泛。

MATLAB代写

MATLAB 是一种用于技术计算的高性能语言。它将计算、可视化和编程集成在一个易于使用的环境中,其中问题和解决方案以熟悉的数学符号表示。典型用途包括:数学和计算算法开发建模、仿真和原型制作数据分析、探索和可视化科学和工程图形应用程序开发,包括图形用户界面构建MATLAB 是一个交互式系统,其基本数据元素是一个不需要维度的数组。这使您可以解决许多技术计算问题,尤其是那些具有矩阵和向量公式的问题,而只需用 C 或 Fortran 等标量非交互式语言编写程序所需的时间的一小部分。MATLAB 名称代表矩阵实验室。MATLAB 最初的编写目的是提供对由 LINPACK 和 EISPACK 项目开发的矩阵软件的轻松访问,这两个项目共同代表了矩阵计算软件的最新技术。MATLAB 经过多年的发展,得到了许多用户的投入。在大学环境中,它是数学、工程和科学入门和高级课程的标准教学工具。在工业领域,MATLAB 是高效研究、开发和分析的首选工具。MATLAB 具有一系列称为工具箱的特定于应用程序的解决方案。对于大多数 MATLAB 用户来说非常重要,工具箱允许您学习应用专业技术。工具箱是 MATLAB 函数(M 文件)的综合集合,可扩展 MATLAB 环境以解决特定类别的问题。可用工具箱的领域包括信号处理、控制系统、神经网络、模糊逻辑、小波、仿真等。

R语言代写问卷设计与分析代写
PYTHON代写回归分析与线性模型代写
MATLAB代写方差分析与试验设计代写
STATA代写机器学习/统计学习代写
SPSS代写计量经济学代写
EVIEWS代写时间序列分析代写
EXCEL代写深度学习代写
SQL代写各种数据建模与可视化代写